Path/Pharm: Coagulation Study Questions

Réussis tes devoirs et examens dès maintenant avec Quizwiz!

A patient is discharged from the hospital with a prescription of warfarin (Coumadin). Which of the following statements indicates successful patient teaching?

"I will avoid herbal remedies." Most commonly used herbs and supplements have a profound effect on drugs for anticoagulation. The patient should never double up on dosing related to a missed dose. The patient should avoid green leafy vegetables due to vitamin K. The patient should not take a multivitamin. (less)

Medications: Vit K antagonist

(warfarin) ALSO A THROMBOLYTIC - Interferes with vitamin K-dependent clotting factors - Levels peak ~90 minutes after administration - Frequent PT and INR monitoring - Variable responses - Counteracted by giving vitamin K

Blood types

- Agglutinogens --> Specific types of antigens on surface of RBCs - ABO blood type classification is based on these agglutinogens --> A, B, AB, O - O- is universal donor, AB+ is universal recipient - If a person receives blood that is a different type it causes a transfusion reaction because antibodies are attacking the foreign cells cause hemolysis --> Med errors most common type of error --> Benadryl

Anemia: lack of bone marrow production of RBCs: Aplastic Anemia

- Bone marrow fails: anemia, leukopenia, and thrombocytopenia - Caused by infection, pancytopenia, radiation, chemicals (benzene), certain drugs or immune disease - Blood transfusions necessary - Treatment: bone marrow transplant HLA match to sibling

What are the two pathways of the coagulation cascade?

1. Intrinsic pathway 2. Extrinsic pathway

Which three substances act to decrease clot formation and dissolve clots?

1. Plasmin 2. Plasminogen 3. Tissue Plasminogen Activator

Please place the steps in order which demonstrate how chronic obstructive pulmonary disease (COPD) can lead to secondary polycythemia.

1. Pulmonary dysfunction 2. Decreased blood oxygen levels 3. Decreased oxygen delivery to kidneys 4. Increase erythropoietin levels 5. Bone marrow stimulation

What is the normal range of hemoglobin for women?

12 to 16 gm/dL

What is the lifespan of RBC's?

120 days

When administering tirofiban, the nurse would administer the drug intravenously over which time frame?

30 minutes Tirofiban is administered intravenously over 30 minutes.

A patient asks the nurse what dose of acetylsalicylic acid (Aspirin) is needed for antiplatelet effects to prevent heart attacks. What dose is most appropriate to reduce platelet aggregation?

300 mg A single dose of 300-600 mg or multiple doses of 30 milligrams inhibit cyclooxygenase in circulating platelets almost completely. The dose of 100 mg is too small. The doses of 625 mg and 1000 mg are too large. (less)

A client who is experiencing the signs and symptoms of an acute myocardial infarction says that his symptoms started at 10:00 AM this morning. It is now 11:15 AM. The nurse understands that the latest time that the client could receive a thrombolytic agent would be which of the following?

4:00 PM Explanation: Treatment with a thrombolytic must be instituted within 6 hours of the onset of symptoms of an acute myocardial infarction to achieve maximum therapeutic effectiveness. In this case, 4 PM would be the latest.

A client is started on enoxaparin immediately after hip surgery. The nurse would explain to the client that this drug will be continued for how long? 1 to 3 days 3 to 7 days 7 to 10 days 10 to 14 days

7 to 10 days When used for hip surgery, enoxaparin typically is administered for 7 to 10 days to prevent deep vein thrombosis that may lead to pulmonary embolism after hip replacement.

What is the normal value of MVC?

80 to 100 femtoliters

What is the normal value of platelets?

90,000 to 450,000 platelets per microliter of blood

Heparin therapy is prescribed to a patient with deep vein thrombosis. Which patient would be at greatest risk of bleeding from this therapy?

A 45-year-old corporate executive with peptic ulcer disease Heparin is also contraindicated in patients with thrombocytopenia, bleeding disorders, and active bleeding other than DIC. This drug should be used with caution in patients with the potential for hemorrhage (e.g., immediately after surgery, with peptic ulcer disease, and with liver disease).

5. A patient is prescribed Warfarin (Coumadin) for the treatment of a blood clot. What is the therapeutic INR range for this medication to be effective?* A. 2-3 B. 1-3 C. 4-8 D. 0.5-2.5

A. 2-3

6. What is an approximate normal range for a PT (prothrombin) level?* A. 10-12 seconds B. 2-3 seconds C. 30-40 seconds D. 60-80 seconds

A. 2-3

A patient states that he heard that he should take an aspirin each day. He wonders whether there is a benefit to doing this and wants to know what the aspirin is supposed to do. Which of the following is an appropriate response by the nurse?

Aspirin decreased the adherence of factors in the blood that can lead to a clot.

3. What is the approximate normal range for an aPTT result?* A. 2-3 second B. 30-40 seconds C. 1.5-2.5 times the normal range D. 10-12 seconds

B. 30-40 seconds

Anemia is present in a patient with laboratory results revealing (NCNC) anemia. Which of the following does the nurse think is the most likely cause? Vitamin B12 deficiency Folic-acid deficiency Iron deficiency Blood loss

Blood loss

10. Your patient's aPTT level is 32 seconds. The nurse would interpret this lab result as?* A. Too high, the patient is at risk for bleeding. B. Too low C. Normal

C. Normal

8. Fill in the blank: Prothrombin turns into _________ with the assistance of clotting factor V.* A. fibrinogen B. fibrin C. thrombin D. vitamin K

C. thrombin

A nurse is preparing to administer an antiplatelet drug. Which medication would the nurse most likely administer? Clopidogrel Alteplase Phytonadione Enoxaparin

Clopidogrel Clopidogrel is an antiplatelet agent. Alteplase is a thrombolytic agent. Phytonadione is an anticoagulant antagonist. Enoxaparin is an LMWH anticoagulant.

A nurse is preparing to administer an antiplatelet drug. Which medication would the nurse most likely administer? Clopidogrel Alteplase Phytonadione Enoxaparin

Clopidogrel Clopidogrel is an antiplatelet agent. Alteplase is a thrombolytic agent. Phytonadione is an anticoagulant antagonist. Enoxaparin is an LMWH anticoagulant.

Which of the following is classified as an antiplatelet agent? (Choose one)

Clopidogrel (Plavix) Plavix is an antiplatelet agent. Activase is a thrombolytic agent. Mephyton is an anticoagulant antagonist. Coumadin is an oral anticoagulant. Lovenox is a parenteral anticoagulant.

Secondary hemostasis requires a stepwise activation of proteins in the blood called __________ _________ that take part in a complex cascade.

Coagulation factors

A client enters the emergency department with symptoms of an ischemic stroke. What

Colon resection 7 days ago Explanation: Thrombolytics are contraindicated when the client has had a recent surgery or traumatic accident because the thrombolytic would disturb clots that have recently formed from the surgery. A history of a recent migraine headache, sprained ankle, or bronchitis would not contraindicate the use of a thrombolytic.

A client with chronic kidney disease is receiving ferrous sulfate. The nurse instructs the client that which finding is a common side/adverse effect associated with this medication? Fatigue Headache Weakness Constipation

Constipation

Both intrinsic and extrinsic pathways go through an array of complex step by step reactions to arrive at a common final pathway which is:

Conversion of prothrombin into thrombin

The nurse is preparing to care for a client with chronic kidney disease and anemia. When planning care, which describes the relationship between chronic kidney disease and anemia? Lack of angiotensin I may cause anemia. Increased production of aldosterone leads to anemia. Anemia is caused by insufficient production of renin. Decreased production of erythropoietin is causing anemia.

Decreased production of erythropoietin is causing anemia. Rationale:Clients with chronic kidney disease do not manufacture adequate amounts of erythropoietin, which is a glycoprotein needed to synthesize red blood cells. Renin, aldosterone, and angiotensin are substances that assist in maintaining blood pressure. Test-Taking Strategy:Focus on the subject, the relationship between chronic kidney disease and anemia. Think about the pathophysiology associated with anemia. Also, knowledge regarding the function of erythropoietin is needed to answer this question. Remember that erythropoietin is a glycoprotein needed to synthesize red blood cells.

Which of the following are NOT commonly associated with bleeding disorders? Purpura Petechiae Decreased prothrombin time Bleeding from the gums Increased activated partial thromboplastin time

Decreased prothrombin time

Clotting disorders: DVT

Deep vein thrombosis - Thrombosis in the femoral vein that can dislodge and travel up inferior vena cava to pulmonary artery and cause an embolism

Please indicate which box in the figure would indicate the step of erythropoietin release.

Erythropoietin stimulates bone marrow

Please indicate which box in the figure would indicate the step of erythropoietin release

Erythropoiten

What does platelet aggregation measure?

Evaluates platelets ability to adhere and form clumps.

A 10-year-old child with hemophilia A has slipped on the ice and bumped the knee. The nurse would prepare to administer which prescription? Injection of factor X Intravenous infusion of iron Intravenous infusion of factor VIII Intramuscular injection of iron using the Z-track method

Factor VIII Rationale:Hemophilia refers to a group of bleeding disorders resulting from a deficiency of specific coagulation proteins. The primary treatment is replacement of the missing clotting factor; additional medications, such as agents to relieve pain, may be prescribed, depending on the source of bleeding from the disorder. A child with hemophilia A is at risk for joint bleeding after a fall. Factor VIII would be prescribed intravenously to replace the missing clotting factor and minimize the bleeding. Factor X and iron are not used to treat children with hemophilia A.

A thrombolytic can be used to treat an acute MI as long as the onset of symptoms occurred within the last 3 hours.

False Treatment with a thrombolytic must be instituted within 6 hours of the onset of symptoms of an acute MI to achieve maximum therapeutic effectiveness.

Blood clots are dissolved by a process termed ____________.

Fibrinolysis

_____________ causes factor Xa inhibition, which inhibits prothrombin from forming thrombin. It produces a predictable anticoagulant effect after administration in fixed doses because it does not bind to plasma protein.

Fondaparinux

A woman believes she has been diagnosed with anemia. Which lab value does not support this diagnosis? Hematocrit of 38% Hemoglobin levels of 18 g/dL Red blood cell total count of 2.6 million per microliter The woman's hematocrit, hemoglobin, and red blood cell count support the diagnosis of anemia

Hemoglobin levels of 18 g/dL --> Hemoglobin levels are elevated which would support t diagnosis of anemia Hematocrit level is low and would support anemia Red blood cell total count of 2.6 million per microliter The woman's hematocrit, hemoglobin, and red blood cell count support the diagnosis of anemia --> not correct because hematocrit is low but the rest are normal

Excessive blood loss is called _________.

Hemolysis

A young patient has an Escherichia coli infection from eating improperly prepared hamburgers. Which of the following is the clinician most worried about the patient developing?

Hemolytic uremic syndrome

Which of the following are treatments for disorders of clotting? Select all that apply. Heparin Vitamin K antagonists Warfarin Aspirin D-dimer

Heparin Vitamin K antagonists Warfarin Aspirin

__________ thrombocytopenia results from the development an antibody to heparin. Some patients develop low platelet count, but the majority develops clots.

Heparin-Induced

What does a complete blood count include?

Hgb, Hct, a number of RBC's, mean corpuscular volume, mean corpuscular hemoglobin, and MCH concentration.

Mr. Hon has been admitted to the hospital and placed on anticoagulant therapy. For what blood-related disorder is this therapy used?

Hypercoagulation Anticoagulants are used to treat hypercoagulation, or excessive coagulation. They prevent the blood from clotting unexpectedly and maintain the flow of blood. Hemophilia is treated with human factor IX complex. Blood cancer is treated with therapies such as chemotherapy or radiation treatment. On the other hand, hemostasis is the process by which the body prevents excessive blood flow during an injury. (less)

A nurse is reviewing clotting factor laboratory values. Activated partial thromboplastin time (aPTT) and international normal ratio (INR) are listed. The nurse questions why the prothrombin time (PT) value may be missing. Which of the following is an appropriate explanation?

INR reflects PT clotting times

____________________ is caused by autoantibodies that develop against platelets.

Immune Thrombocyopenic Purpura

A child, following a viral infection, presents with thrombocytopenia. Which is the most likely type of thrombocytopenia?

Immune thrombocytopenia purpura

A patient has aplastic anemia with pancytopenia. Which of the following can the nurse expect? Select all that apply.

Increase risk for infection Anemia Increased risk for bleeding

A sickle cell anemia crisis leads to hemolysis of red blood cells (RBCs). Which of the following may the nurse expect to see?

Increased reticulate percentage

A sickle cell anemia crises leads to hemolysis of red blood cells (RBCs). Which of the following may the nurse expect to see? Decreased serum billirubin Increased reticulocyte percentage Increased hematocrit Decreased likelihood of jaundice

Increased reticulocyte percentage --> reticulocytes are immature RBCs, and levels will increase to replace lost cells

A nurse is caring for a patient receiving warfarin drug therapy. The patient informs the nurse that he is also taking chamomile, which is an herbal remedy. What effect of the interaction between warfarin and the herb should the nurse inform the patient about?

Increased risk for bleeding The nurse should inform the patient about the increased risk for bleeding, which is an effect of the interaction between warfarin and the herb. Decreased effectiveness of chamomile, increased absorption of warfarin, and increased risk for hypertension are not effects of interaction between warfarin and chamomile. (less)

A patient has aplastic anemia with pancytopenia. Which of the following can the nurse expect? Select all that apply. Increased risk for infection Anemia Increased risk for bleeding Elevated hematocrit Polycythemia

Increased risk for infection Anemia Increased risk for bleeding

The _________ pathway is stimulated by tissue damage incurred by injury to endothelial lining of a blood vessel.

Intrinsic

What term would be most applicable for this person's skin tone, which has a more yellow-like appearance? Scleritis Jaundice Hyperpigmentation Melasma

Jaundice

What does PT measure?

Measure time to clot, evaluates the extrinsic pathway of coagulation cascade

What does D-Dimer measure?

Measures fibrin degradation products.

What is the reticulocyte count?

Measures the number of NEW RBC's in the blood and helps determine whether the bone marrow is producing new red cells at an appropriate rate.

Explain Iron Deficiency Anemia.

Most common cause of anemia worldwide. Increased risks include women of child bearing age, infants and children, vegetarians, elderly adults, and individuals with GI bleeding. Etiology: Inadequate intake, excessive menstrual blood loss, and GI blood loss. Pathophysiology: Iron is needed in the diet to synthesize Hgb. The heme portion of Hgb contains iron that carries oxygen atoms. Lack of sufficient iron leads to poor oxygen transport by iron deficient Hgb molecules. Clinical Presentation: Age, current medications, history of GI symptoms, current menstrual history, use of NSAIDs and aspirin, frequency of alcohol use and smoking, asked about dark stool. Signs and symptoms: Fatigue, weakness, exercise intolerance, hair loss, cheilitis, glossitis, nail changes, craving of nonfood substances (pica) Diagnosis: CBC, peripheral smear, serum iron, TIBC, serum ferritin, chronic lead poisoning. Treatment: Oral iron therapy, parenteral iron therapy

Explain Sickle Cell Anemia.

Most common inherited hemoglobinopathies. Etiology: Genetic mutation that directs abnormal synthesis of the Hgb molecule. Transmitted by a recessive trait. Clinical presentation: Fatigue and exercise intolerance, high bilirubin, severe pain, fever, tachycardia, anxiety, chest pain, hand-foot syndrome. Diagnosis: Blood sample, CBC, peripheral blood smear, reticulocytosis indicates a high amount of hemolysis. Treatment: Oxygen, hydration, pain medications, prophylactic antibiotics, folic acid supplements, bone marrow transplant, gene therapy, nitric acid.

A 75-year-old client presents to the physician's office with bleeding gums and multiple bruises. When the nurse reviews the client's drug history, the nurse finds that he is prescribed aspirin 81 mg/d. What drug may cause increased bleeding when used in conjunction with the aspirin?

NSAIDs NSAIDs, which are commonly used by older adults, also have antiplatelet effects. Clients who take an NSAID daily may not need low-dose aspirin for antithrombotic effects.

Regular use of antiplatelet medications can contribute to increased risk of bleeding. Your patient, a 79-year-old woman, is admitted to the hospital with black, tarry stools and several large, scattered ecchymosis. She has a history of TIAs and has taken clopidogrel for several years. What would you expect the physician to prescribe to control her bleeding?

None of these Clopidogrel irreversibly blocks the (ADP) receptor on platelet cell membranes. This effectively prevents platelet aggregation for the lifespan of the platelet. Platelet transfusion should be considered in patients who take these drugs and develop bleeding complications. (less)

A trauma patient arrives in the emergency department with a stab wound. If a blood sample were to be looked at underneath the microscope, which type of anemia would likely be present?

Normocytic

Explain Hemolytic Anemia.

Occurs when erythrocyte destruction outpaces RBC synthesis by the bone marrow. Signs and Symptoms: Fatigue, pallor, shortness of breath, and tachycardia, chills, jaundice, dark urine, enlarged spleen. Diagnosis: CBC and bone marrow response, reticulocyte count, peripheral smear, Hgb electrophoresis, bone marrow examination, tests for the presence of autoantibodies. Treatment: Folic acid and iron replacement, corticosteroids, transfusions.

A client is to receive clopidogrel. The nurse would expect to administer this agent by which route? Oral Intravenous Intramuscular Subcutaneous

Oral Clopidogrel is administered orally.

A client is receiving heparin. Which of the following would the nurse use to monitor the effects of the drug?

Partial thromboplastin time Heparin's effectiveness is monitored by the results of the partial thromboplastin time. The INR and PT are used to monitor warfarin. Vitamin K is the antidote for warfarin, and levels are not monitored to evaluate the effects of any anticoagulant. (less)

The nurse analyzes the laboratory results of a child with hemophilia. The nurse understands that which result will most likely be abnormal in this child? Platelet count Hematocrit level Hemoglobin level Partial thromboplastin time

Partial thromboplastin time Rationale:Hemophilia refers to a group of bleeding disorders resulting from a deficiency of specific coagulation proteins. Results of tests that measure platelet function are normal; results of tests that measure clotting factor function may be abnormal. Abnormal laboratory results in hemophilia indicate a prolonged partial thromboplastin time. The platelet count, hemoglobin level, and hematocrit level are normal in hemophilia. Test-Taking Strategy:Focus on the subject, laboratory tests used to monitor hemophilia, and note the strategic words, most likely. Recalling the pathophysiology associated with this disorder and recalling that it results from a deficiency of specific coagulation proteins will direct you to the correct option.

Laboratory values reveal the following: hemoglobin of 19 grams per deciliter and hematocrit of 53%. Which of the following correctly interprets these values? Select all that apply.

Percentage of blood volume that is plasma is reduced. Both hemoglobin and hematocrit levels are elevated.

In ___________ hemostasis platelets form a fundamental platelet plug.

Primary

What are the two types of thrombocytosis?

Primary and secondary

CH 13 Chapter Summary

RBCs are synthesized in the bone marrow in response to EPO secretion by the kidneys. • RBCs are nonnucleated cells that carry oxygen on an Hgb molecule. Iron is necessary to form Hgb. • Reticulocytes are immature RBCs that are produced in high numbers by active bone marrow. • Bilirubin is a breakdown product of RBCs. Jaundice occurs when there is a high amount of bilirubin in the bloodstream. • The spleen is the graveyard of RBCs. Splenomegaly and splenic dysfunction can occur if a constant number of RBCs need to be processed by the spleen. • Anemias are disorders in which there is a lack of mature, healthy RBCs, leading to impaired oxygen delivery to cells and tissues. • Causes of anemia include blood loss, accelerated destruction of erythrocytes, and failure of erythrocyte production. • Common symptoms of anemia include fatigue, dyspnea, exercise intolerance, pallor, and tachycardia. • Diagnosis of anemia begins with a CBC, which establishes the presence of the disorder. The indices MCV, MCH, and MCHC provide information about possible causes. • The MCV index indicates the size of the RBC. Normal MCV indicates normal-sized RBCs. Low MCV indicates small RBCS called microcytic cells. High MCV indicates megaloblastic or macrocytic cells. The MCHC indicates the color of the RBCs. Normal MCHC indicates normal color of cells. Low MCHC indicates pale cells. • Anemias can be categorized according to how they appear on a peripheral blood smear. The various descriptive categories include normocytic normochromic (NCNC), microcytic hypochromic, or megaloblastic (also called macrocytic). • A large amount of blood loss causes NCNC anemia. • Iron deficiency is the most common cause of anemia. Menorrhagia and GI bleeding are the most common causes. • Iron deficiency is caused by a slow loss of RBCs from the body. With this slow loss, the body gradually loses Hgb, which is the main source of iron in the body and therefore causes iron deficiency. • Iron deficiency causes a microcytic, hypochromic anemia. • Iron deficiency in a male or post-menopausal female should increase the clinician's suspicion of GI bleeding due to ulcer or colon cancer. • Serum ferritin, transferrin, TIBC, and FOBT are tests needed in iron deficiency. • Iron deficiency in adult males and postmenopausal females requires colonoscopy to assess for colon cancer. • Vitamin B12 deficiency causes megaloblastic anemia and neurologic dysfunction of the spinal cord. Older adults can develop gait dysfunction, sensory loss in the feet, and dementia-like symptoms. • Pernicious anemia is a vitamin B12 deficiency caused by autoimmune destruction of IF in the stomach. Vitamin B12 requires IF for absorption. Therefore, pernicious anemia is best treated with parenteral vitamin B12. • Folic acid deficiency also causes megaloblastic anemia. This deficiency allows accumulation of homocysteine, an amino acid that causes endothelial damage leading to arteriosclerosis. • Folic acid deficiency in pregnant women can cause spinal cord and CNS abnormalities in the fetus. • SCA and thalassemia are the most common disorders of Hgb structure (termed hemoglobinopathy). • Vaso-occlusive crises can occur when individuals with SCA endure hypoxia, stress, or infection. Children often suffer numerous vaso-occlusive crises as they grow. • Vaso-occlusive crises are severely painful episodes that often occur in the long bones or organs. Opiate medications are commonly needed for episodes, which can lead to dependence. • Vaso-occlusive crises can lead to ischemia of an organ. Ischemia of the brain can cause stroke. • Transfusion, a mainstay of treatment in anemia, is commonly used when Hgb is 7 g/dL. • Type O blood is the universal donor; type AB is the universal recipient. • HDN commonly occurs at birth due to ABO incompatibility of the mother and infant when a small quantity of blood from the mother can enter fetal circulation. The maternal antibodies destroy some infant RBCs. This causes a mild anemia with jaundice in the infant. • Secondary polycythemia is stimulated by chronic hypoxia of the bloodstream. • Polycythemia vera is a cancerous condition of the blood cells that causes excessive accumulation of RBCs, WBCs, and platelets in the bloodstream. • Bone marrow transplantation requires a donor with matching tissue type of the recipient.

What are the only cells that carry oxygen?

Red Blood Cells

An immature RBC is called a __________. It is approximately 1% of the RBC population.

Reticulocyte

Which of the following patient is most at risk for erythroblastosis fetalis? Rh-positive mother with her third pregnancy Rh-negative mother with her first pregnancy Rh-negative mother with her third pregnancy Rh-positive mother with her first pregnancy

Rh-negative mother with her third pregnancy

___________ thrombocytosis is an elevated platelet count caused by another primary condition, such as iron deficiency anemia, cancer, inflammation, infection, surgery, or myeloproliferative disorders. The excessive number of platelets does not cause clotting.

Secondary

What is the graveyard of the RBC's and an organ of immunity?

Spleen

A patient has hemophilia. Which shows a correct understanding of this disease?

Thalassemia is a result of a genetic mutation.

A stable daily dose of warfarin is reached when which parameter is achieved?

The PT and INR are within their therapeutic ranges, and the dose does not cause bleeding. When warfarin is started, PT and INR should be assessed daily until a stable daily dose is reached (the dose that maintains PT and INR within therapeutic ranges and does not cause bleeding). (less)

What are platelets?

The blood cells that cause clotting of the blood.

The client is diagnosed with hepatitis A, diabetes type 1, and portal hypertension. He develops a DVT, and the physician prescribes warfarin. The nurse is concerned, for what reason?

The client is more likely to experience bleeding. Warfarin is more likely to cause bleeding in clients with liver disease because of decreased synthesis of vitamin K. In addition, warfarin is eliminated only by hepatic metabolism and may accumulate with liver impairment. (less)

Which of the following would the nurse identify as the end of the intrinsic pathway?

Thrombin formation

__________ is the generation of an occlusive thrombus that obstructs blood flow in an artery or vein.

Thrombosis

When absorbed into the bloodstream from the GI system, iron is transported by a protein called _________.

Transferrin

True or false: Women of childbearing age are at higher risk than men for anemia.

True

Warfarin typically takes 3 days to achieve its onset of action.

True Warfarin's onset of action is about 3 days; its effects last for 4 to 5 days.

Medication: Thrombolytics

Urokinase Indications - Acute MI, PE, ischemic stroke Therapeutic effects - Converts plasminogen to plasmin which breaks down fibrin clots, fibrinogen and other plasma proteins - Lysis thrombi and emboli Adverse effects - Headache - Angioneurotic edema - Hypotension - Skin rash - Bleeding - Breathing difficulties/bronchospasm - Pain - Fever - Anaphylactic

What are the three major steps of hemostasis?

Vasoconstriction, development of a platelet plug, and blood coagulation

A patient who is receiving warfarin (Coumadin) has blood in his urinary drainage bag. What medication will likely be ordered by the physician?

Vitamin K Correct Explanation: Vitamin K is the antidote for warfarin overdose. Aminocaproic acid is used to control excessive bleeding from systemic hyperfibrinolysis. Platelets are a blood product, not a medication. Protamine sulfate is the antidote for heparin therapy. (less)

Parents have just been told that their child has hemophilia. Which of the following information would be correct for the nurse to share?

Your child has an increased risk of bleeding, so care must taken even with small wounds.

A patient is on anticoagulation therapy. Which factors are likely to be elevated? Select all that apply.

aPTT, PT, INR

Petechiae

bleeding disorder

Alcohol abuse and liver damage

clotting disorder

Malignant tumors and high estrogen

clotting disorder

The intrinsic pathway is also known as the:

contact activation pathway

A nursing student moves from sea level to mile-high elevation. Please place in the proper order the sequence of events that likely occurred for her blood laboratory values. decreased arterial oxygen increased erythropoietin release bone-marrow stimulation increased RBC synthesis Increased Hct

decreased arterial oxygen increased erythropoietin release bone-marrow stimulation increased RBC synthesis Increased Hct

Where is iron absorbed?

duodenum and upper jejunum

A 35-years-old patient's symptoms and findings include shortness of breath and pale skin, along with an elevated activated partial thromboplastin time (aPTT) value. Elevated aPTT indicates_________. increased clot formation increased clotting time for the intrinsic pathway increased fibrinogen breakdown decreased reticulocyte count increased prothrombin time

increased clotting time for the intrinsic pathway

Warfarin (Coumadin)

oral anticoagulant

Please place the steps in order which demonstrate how chronic obstructive pulmonary disease (COPD) can lead to secondary polycythemia. ulmonary dysfunction decreased blood oxygen levels decreased oxygen deliver to kidneys increased erythropoietin level bone-marrow stimulation

pulmonary dysfunction decreased blood oxygen levels decreased oxygen deliver to kidneys increased erythropoietin level bone-marrow stimulation

What are the difficulties encountered when the physician orders lepirudin for management of atrial fibrillation? (Select all that apply.)

• No antidote is available. • Must be given IV • Requires frequent lab tests Explanation: This drug is not suitable for long-term treatment because it must be given IV and no antidote is available in the event of an overdose or if bleeding occurs. Its duration of action is 24 hours. It requires frequent lab tests that might not be readily available. It may cause anxiety, not depression.

Polycythemia

→ opposite of anemia - abundance of RBCs

What is the mean corpuscular hemoglobin (MCH)?

It is the average amount of hub in an average RBC. Normal is 27-32 picograms.

What is red blood cell distribution?

It measures the variation in RBC size and width.

What is the oxyhemoglobin dissociation curve?

It represents the relationship between PO2, and the affinity of Hgb to oxygen.

The home care nurse is making a monthly visit to a client with a diagnosis of pernicious anemia who has been receiving a monthly injection of cyanocobalamin. Before administering the injection, the nurse evaluates the effects of the medication and determines that a therapeutic effect is occurring if the client makes which statement? "I feel really light-headed." "I no longer have any nausea." "I have not had any pain in a month." "I feel stronger and have a much better appetite."

"I feel stronger and have a much better appetite." Rationale:Cyanocobalamin is essential for DNA synthesis. It can take up to 3 years for the vitamin B12 stores to be depleted and for symptoms of pernicious anemia to appear. Symptoms can include weakness, fatigue, anorexia, loss of taste, and diarrhea. To correct deficiencies, a crystalline form of vitamin B12, cyanocobalamin, can be given intramuscularly. The client statements in options 1, 2, and 3 do not identify a therapeutic effect of the medication. Test-Taking Strategy:Focus on the subject, therapeutic effects of cyanocobalamin, and note the words therapeutic effect is occurring. This will assist in eliminating option 1. Regarding the remaining choices, noting that the medication is a vitamin will direct you to the correct option.

The nurse has provided instruction to a client with chronic kidney disease who has a prescription for epoetin alfa. Which statement by the client indicates that teaching was effective? "I have to receive this medication subcutaneously." "I will receive this medication through intramuscular injection." "This medication has to be administered using the Z-track method." "I will take this medication orally with the rest of my morning pills."

"I have to receive this medication subcutaneously." Rationale:Epoetin alfa is administered parenterally by the intravenous or subcutaneous route. It cannot be given orally because it is a glycoprotein and would be degraded in the gastrointestinal tract. Test-Taking Strategy:Note the strategic word, effective. Focus on the subject, epoetin alfa. Knowledge regarding the administration of this medication is required to answer this question. Eliminate options 2 and 3 first because they are comparable or alike. Z-track is an intramuscular injection. From the remaining options, remember that epoetin alfa (erythropoietin) is administered parenterally by either the intravenous or the subcutaneous route.

The nurse has provided medication instructions to a client with an iron deficiency anemia who will be taking iron supplements. Which statement made by the client indicates an understanding of this medication? "I need to increase my fluid intake." "I need to eliminate fiber foods from my diet." "I need to take the medication with water before a meal." "I need to be sure to chew the tablet thoroughly before swallowing it."

"I need to increase my fluid intake." Rationale:Iron preparations can be very irritating to the stomach and are best taken between meals. Because iron supplements may be associated with constipation, the client would increase fluids and fiber in the diet to counteract this side effect of therapy. Iron preparations need to be taken with a substance that is high in vitamin C to increase its absorption. The tablet is swallowed whole and not chewed. Test-Taking Strategy:Focus on the subject, client understanding of how to take iron supplementation. Recall that constipation is a side effect of iron supplements; then select the option that is a countermeasure.

A client is discharged from the hospital with a prescription of warfarin. Which statement indicates successful client teaching? "If I miss a dose, I will take two doses." "I will avoid herbal remedies." "I will eat spinach or broccoli daily." "I will discontinue my other medications."

"I will avoid herbal remedies." Most commonly used herbs and supplements have a profound effect on drugs for anticoagulation. The client should never double up on dosing related to a missed dose. The client should avoid green leafy vegetables due to vitamin K. The client should not discontinue his or her medications without first consulting with the primary health care provider.

The nurse is preparing a client with thrombocytopenia for discharge. Which statement by the client about measures to minimize injury indicates that discharge teaching was effective? Select all that apply. "I will not blow my nose if I get a cold." "I may continue to use an electric shaver." "I need to use an enema instead of laxatives for constipation." "I definitely will play football with my friends this weekend." "I would use a soft-bristled toothbrush to avoid mouth trauma."

"I will not blow my nose if I get a cold." "I may continue to use an electric shaver." "I would use a soft-bristled toothbrush to avoid mouth trauma." Rationale:Bleeding precautions are used to protect the client with thrombocytopenia from bleeding. The client with thrombocytopenia may experience internal and external bleeding. Bleeding is frequently provoked by trauma, but it also may be spontaneous. The client with thrombocytopenia needs to be educated about activities that increase the risk for bleeding, such as contact sports and trauma to oral, nasal, and rectal mucosa. This will help to eliminate options 3 and 4. Test-Taking Strategy:Note the strategic word, effective. Focus on the subject, thrombocytopenia. Recalling that thrombocytopenia is a disorder in which the client has a decrease of thrombocytes, commonly known as platelets, and recalling the function of platelets will help you to answer this question.

The nurse educates a client diagnosed with atrial fibrillation about the prescribed warfarin. Which client statements establish the need for further clarification? "I will keep my lab appointments for prothrombin and INR levels." "I will take the warfarin at the same time each day." "I will not eat green leafy vegetables, broccoli, yogurt, or cheese." "I will not drink alcohol unless it is all right with my primary care provider."

"I will not eat green leafy vegetables, broccoli, yogurt, or cheese." The nurse needs to clarify teaching regarding the intake of foods rich in Vitamin K. The client should not avoid eating foods with Vitamin K, but eat consistent amounts every day so therapeutic levels for the warfarin may be maintained. The client has an adequate understanding of the importance of keeping lab appointments for PT and INR levels used to monitor and regulate the warfarin dosage. The client should take the warfarin the same tie each day to maintain consistent blood levels of the drug. The client should not drink alcohol unless the primary care provider approves it, and instructs the client as to how much and how often. Alcohol will increase the risk for bleeding.

Epoetin alfa is prescribed for a client diagnosed with chronic kidney disease. The client asks the nurse about the purpose of the medication. Which response by the nurse is most appropriate? "It is used to treat anemia." "It is used to lower your blood pressure." "It will help to increase the potassium level in your body." "It is an anticonvulsant medication given to all clients after dialysis to prevent seizure activity."

"It is used to treat anemia."

A client is receiving low-molecular-weight heparin to prevent thromboembolic complications. The nursing student asks the instructor the reason why this treatment is given instead of heparin. What is the instructor's best explanation of the rationale for LMWH over heparin? "LMWH is associated with less thrombocytopenia than standard heparin." "LWMH is associated with stronger anticoagulant effects than standard heparin." "LWMH is given to clients who have a history of blood dyscrasia." "LWMH is more effective than standard heparin for clients with hypertension."

"LMWH is associated with less thrombocytopenia than standard heparin." Low-molecular-weight heparins are associated with less thrombocytopenia than standard heparin. Low-molecular-weight heparin is not stronger than standard heparin. Low-molecular-weight heparin is administered cautiously in clients with blood dyscrasia and hypertension.

A patient is receiving low-molecular-weight heparin to prevent thromboembolic complications. The nursing student asks her instructor the reason why this treatment is given instead of heparin. What is the instructor's best explanation of the rationale for LMWH over heparin?

"LMWH is associated with less thrombocytopenia than standard heparin." Low-molecular-weight heparins are associated with less thrombocytopenia than standard heparin. Low-molecular-weight heparin is not stronger than standard heparin. Low-molecular-weight heparin is administered cautiously in patients with blood dyscrasia and hypertension. (less)

Your 86-year-old client, who was admitted earlier in the week with thrombophlebitis, is being sent home on enoxaparin. Which of the following statements by your client suggests that he understands proper management of his condition and proper use of the drug?

"The medication will increase my risk of bleeding." Since antiplatelet and anticoagulant drugs increase the risk of bleeding, clients taking such drugs should take precautions to avoid injury. Clients should report any sign of bleeding to a health care professional. (less)

Which client statement demonstrates an accurate understanding of the action of a prescribed anticoagulant? "This medication will keep unnecessary clots from forming in my blood." "If a big clot starts to form, this medication will help break it up." "Anticoagulants thin the blood so clots can't get as big." "If I have another stroke, this medication will bust up the clot before too much damage is done."

"This medication will keep unnecessary clots from forming in my blood." An anticoagulant slows the body's normal blood-clotting processes to prevent harmful blood clots from forming. This type of drug is often called a "blood thinner"; however, it cannot dissolve any clots that have already formed and does not make your blood thin.

A patient is being administered heparin IV and has been started on warfarin (Coumadin). The patient asks the nurse why she is taking both medications. What is the nurse's most accurate response?

"Warfarin takes 3-5 days to develop anticoagulant effects, and you still need heparin." Anticoagulant effects do not occur for 3-5 days after warfarin is started because clotting factors already in the blood follow their normal pathway of elimination. The statement "After a certain period of time, you must start warfarin and heparin together" does not explain clearly the reason for the two medications concurrently. The statement "You will need both warfarin and heparin for several days" does not explain clearly the reason for the two medications. The statement "Warfarin cannot be given without heparin due to the amount of clotting you need" is not accurate. (less)

Blood components: Hct

- % of whole blood consisting of RBCs - Men = 45-52%, women 37-48%

Blood components: Hgb

- 95% of oxygen travels attached to RBC - Normal HgbA, fetal HgbF - Males 13-18, female 12-16 - Anemia = <13 in men, <12 in women - Metabolized to biliverdin - colors urine, faces - biliverdin breaks down to bilirubin which causes jaundice

Coagulation

- Calcium and Vit K needed - Two pathways → trigger coagulation -> Intrinsic - damage to vessel → injury to vessel can be activated by stasis of blood - AFib ------- APTT - clotting time - 25-40 seconds -> Extrinsic - damage outside vessel ------- Clotting time is PT and INR - PT is 10-14 seconds; INR is 1 ------- Comes into contact with tissue to form tissue factor VII leading to activation of factor X - Both pathways terminate at the same step - Activation of factor X - Convert prothrombin to thrombin - Convert fibrinogen to fibrin

Anemia: Deficiencies

- Deficiencies - iron, b12, folic acid -------- Iron deficiency - slow over time person might not be aware -------- Pernicious anemia: vitamin B12 is not absorbed ---------------- Treatment: B12 injections ------- Megaloblastic anemia: folic acid deficiency --------------- At risk: pregnant and lactating women, alcoholics, elderly, celiac disease --------------- Important to note both folic acid levels and vitamin B12 levels with megaloblastic anemia (as adequate B12 needed for successful folate metabolism) --------------- Treatment: Oral replacement of folic acid - 1 mg per day -------------- Foods containing folic acid: whole grains, beans, leafy vegetables like spinach -

Defection Coagulation and impaired synthesis

- Deficiencies in clotting factors -------- Hemophilia → inherited disorders, increased bruising etc. WILL BLEED OUT OK MICHEEEEALL -------- Von willebrand disease (which is not pronounced how Michael said) → SAME Impaired synthesis → Vit K fat soluble produced by intestinal bacteria and pts on broad spectrum antibiotics have impaired fat absorption and Vit K is not produced and effects coagulation ------ Liver disease -------Vit K levels ------------ Vitamin k is produced in the intestinal bacteria - infants are not born with it so it is given as a shot to limit risk to hemorrhage

Medications for anemia

- Epoetin Alfa - for renal failure and to decrease need for blood transfusions - Darbapoentin Alfa for chronic renal failure - Methoxy Polyethylene Glycol Epoetin Beta for renal failure - Ferrous sulfide - Hydroxyurea

Anemia: hemoglobinpathies

- Hemoglobinophathies - inherited disorders with abnormal production or structure of the hemoglobin molecule ex. Sickle cell ex. Sickle cell - avoid triggers (stress, infection), hydroxyurea (medication used in sickle cell), blood transfusion, bone marrow transplant -------- Hydroxyurea treats cancer - stops growth of cells and can be used for sickle cell anemia - helps prevent the sickle shape of the cell and makes RBCs bigger s/s - hypoxia, dehydration, pain, enlarged spleen

Blood components: Iron

- Iron is required for RBC formation, specifically Hgb - Iron from RBCs are recycled - Absorbed in small intestine - Transported in the blood bound to transferrin - Used to treat for iron deficiency anemia or for those receiving treatment with EPO-A - Menhorrhagia, GIB, inadeqyate iron - Oral ferrous sulrate - if they ingested lead chelation therapy

Polycythemia - Secondary

- Secondary aka erythrocytosis - More common than primary form - Caused by prolonged hypoxia i.e. COPD - Symptoms develop slowly, spleen may enlarge - EPO LEVELS ARE HIGH - TRYING TO STIMULATE RBCS CAUSE BODY THINKS YOU ARE HYPOXIC AND COMPENSATED Correcting underlying hypoxia will reverse the condition

Hemeostasis

- Stopping of bleeding - Two main processes --> Primary hemostasis - platelet aggregation --> Secondary hemostasis - fibrin formation which produces a clot called a thrombus - Three major steps in hemostasis - vasoconstriction, platelet plug formation, blood coagulation - Excessive bleeding, excessive clotting --> Thrombus clot formation can be in response to stagnant/sluggish blood flow - viscous ---> Thrombosis: Generation of an occlusive clot ---> Clot formation is balanced by clot dissolution →fibrinolysis

Clot dissolution aka fibrinolysis

- TPA tissue plasminogin activator - Converts plasminogen to plasmin - Plasmin breaks down clots - STUFF BREAKS CLOTS

Lack of bone marrow production of EBCs: Lack of EPO

- Usually due to kidney failure, hypothyroidism may also decrease EPO - Recombinant EPO can be given

Clotting Disorders: AFib

- Weakened pumping of the blood in the atrium can lead to venous stasis in lower extremities and cause DVTs and cause strokes

What is the normal range of hemoglobin for men?

13 to 18 gm/dL

The nurse is evaluating the results of laboratory studies for a client receiving epoetin alfa. When would the nurse expect to note a therapeutic effect of this medication? Immediately After 3 days of therapy After 1 week of therapy After 2 weeks of therapy

2 weeks Rationale:Epoetin alfa stimulates erythropoiesis. It takes 2 to 6 weeks after initiation of therapy before a clinically significant increase in hematocrit is observed. Therefore, this medication is not intended for clients who require immediate correction of severe anemia, and it is not a substitute for emergency blood transfusions. Test-Taking Strategy:Focus on the subject, therapeutic effects of epoetin alfa. Use knowledge about anatomy and physiology. Knowing that the medication stimulates bone marrow production of red blood cells (erythropoiesis) will assist in directing you to the correct option.

What is the normal range of hematocrit for women?

37% to 48%

What is polycythemia?

A disorder characterized by over proliferation of all blood cells in the bone marrow.

What is a thrombus?

A durable blood clot

A paramedic assesses a trauma patient and observes the following: alert and conscious, heart rate (HR) of 92 beats per minute (bpm), blood pressure 128/80, with elevated respiratory rate. Based on this information, what percentage of blood volume does the paramedic believe the patient has lost? < 15% 15% to30% 30% to 40% > 40%

<15%

What is the normal range of hematocrit for men?

45% to 52%

A client with liver dysfunction has low serum levels of fibrinogen and a prolonged prothrombin time (PT). Based on these findings, which actions would the nurse plan to promote client safety? Select all that apply. Your Answers: 1Monitor serum potassium levels. 2Weigh client daily, and monitor trends. 3Monitor for symptoms of fluid retention. 4Provide the client with a soft toothbrush. 5Instruct the client to use an electric razor. 6Monitor all secretions for frank or occult blood.

4Provide the client with a soft toothbrush. 5Instruct the client to use an electric razor. 6Monitor all secretions for frank or occult blood. Rationale:Fibrinogen is produced by the liver and is necessary for normal clotting. A client who has insufficient levels is at risk for bleeding. The PT is prolonged when one or more of the clotting factors (II, V, VII, or X) is deficient, so the client's risk for bleeding is also increased. A soft toothbrush, an electric razor, and monitoring secretions for evidence of bleeding are measures that provide for client safety. Focus on the subject, low serum levels of fibrinogen and a prolonged PT. Specific knowledge of the substances produced by the liver is needed to answer this question, as well as knowledge of laboratory abnormalities found in liver dysfunction. Eliminate options 1, 2, and 3 because these actions are directed toward fluid and electrolyte disturbances that can occur with liver dysfunction.

Laboratory studies are performed for a client suspected to have iron-deficiency anemia. The nurse reviews the laboratory results, knowing that which result indicates this type of anemia? 1Elevated hemoglobin level 2Decreased reticulocyte count 3Elevated red blood cell count 4Red blood cells that are microcytic and hypochromic

4Red blood cells that are microcytic and hypochromic Rationale: In iron-deficiency anemia, iron stores are depleted, resulting in a decreased supply of iron for the manufacture of hemoglobin in red blood cells. The results of a complete blood cell count in clients with iron-deficiency anemia show decreased hemoglobin levels and microcytic and hypochromic red blood cells. The red blood cell count is decreased. The reticulocyte count is usually normal or slightly elevated. Test-Taking Strategy:Focus on the subject, laboratory findings. Eliminate options 1 and 3 first, knowing that the hemoglobin and red blood cell counts would be decreased. From the remaining options, select the correct option over option 2 because of the relationship between anemia and red blood cells.

A client enters the hospital emergency department with a nosebleed. On assessment, the client tells the nurse that the nosebleed just suddenly began. The nurse notes no obvious facial injury. Which is the initial nursing action? 1Insert nasal packing. 2Prepare a nasal balloon for insertion. 3Place the client in a semi-Fowler's position, and apply ice packs to the nose. 4Sit the client down, ask the client to lean forward slightly, and apply pressure to the nose for 5 to 10 minutes.

4Sit the client down, ask the client to lean forward slightly, and apply pressure to the nose for 5 to 10 minutes. Rationale:The initial nursing action for a client with a nosebleed is to sit the client down, ask the client to lean forward slightly, and apply pressure to the nose for 5 to 10 minutes. Inserting nasal packing or preparing a nasal balloon is not an appropriate initial intervention. These interventions are used when conservative measures fail. Placing the client in a semi-Fowler's position would promote swallowing blood, which is not helpful because of the risk of vomiting and resultant aspiration. Test-Taking Strategy:Note the strategic word, initial. Eliminate options 1 and 2 first, knowing that initial treatment is always directed at a conservative measure. Attempt to visualize each of the remaining options. Note that the correct option addresses the application of pressure.

The nurse is reviewing the coagulation studies of a client who is receiving a heparin infusion. The client's baseline partial thromboplastin time is 32 seconds. Which result would indicate therapeutic effectiveness?

64 seconds Therapeutic range for heparin would be 1.5 to 2.5 times the client's baseline. For a baseline value of 32 seconds, this would range from 48 to 80 seconds. A value of 64 seconds would be considered therapeutic. (less)

The nurse is reviewing the coagulation studies of a client who is receiving a heparin infusion. The client's baseline partial thromboplastin time is 32 seconds. Which result would indicate therapeutic effectiveness? 32 seconds 40 seconds 64 seconds 96 seconds

64 seconds Therapeutic range for heparin would be 1.5 to 2.5 times the client's baseline. For a baseline value of 32 seconds, this would range from 48 to 80 seconds. A value of 64 seconds would be considered therapeutic.

What is the normal lifespan of a platelet?

7 to 10 days

A client is started on enoxaparin immediately after hip surgery. The nurse would explain to the client that this drug will be continued for how long?

7 to 10 days When used for hip surgery, enoxaparin typically is administered for 7 to 10 days to prevent deep vein thrombosis that may lead to pulmonary embolism after hip replacement. (less)

A nurse will use extreme caution when administering heparin to which of the following patients?

A 38-year-old male with peptic ulcer disease Heparin should be administered with extreme caution to patients with peptic ulcer or liver disease or after surgery because those patients would have greater risk for hemorrhage or excessive blood loss. Urticaria is a listed adverse effect of taking heparin and would therefore be expected. A patient who takes heparin would be advised to not drink heavily or smoke, but neither would have the safety risk posed by a patient with peptic ulcer. A patient with an accelerated heart rate and on heparin therapy would be at no special risk. (less)

Which of the following patients would likely be the best candidate to receive alteplase recombinant therapy?

A 68-year-old male who has had an ischemic stroke that resulted in neurological deficits Alteplase recombinant is the only drug approved for treating ischemic stroke. After a CT scan rules out intracranial bleeding or hemorrhagic stroke, treatment should be initiated within 3 hours after the onset of the stroke. The drug can be used only for patients who have a clinical diagnosis of stroke with clinically meaningful neurologic deficit because its adverse effects can be fatal. Alteplase recombinant may also be used to treat massive pulmonary embolism, but only in patients who are at least 18 years old because the drug's safety and efficacy for children have not been established.

The nursing student is presenting a clinical conference and discusses the cause of β-thalassemia. The nursing student informs the group that a child at greatest risk of developing this disorder is which of these? A child of Mexican descent A child of Mediterranean descent A child whose intake of iron is extremely poor A breast-fed/chest fed child of a parent with chronic anemia

A child of Mediterranean descent Rationale:β-Thalassemia is an autosomal recessive disorder characterized by the reduced production of one of the globin chains in the synthesis of hemoglobin (both parents must be carriers to produce a child with β-thalassemia major). This disorder is found primarily in individuals of Mediterranean descent. Options 1, 3, and 4 are incorrect.

Explain Anemia of Acute Blood Loss?

A precipitous drop in the RBC population caused by hemorrhage. Risk Factors: Recent trauma, abdominal trauma, head trauma, history or aortic aneurysm, anticoagulants, hematologic neoplasms, family history of hereditary blood disorders, alcohol abuse, ectopic pregnancy, miscarriage. Etiology: Rapid loss of blood as in hemorrhage caused by trauma, childbirth, or rupture of a major blood vessel or organ Pathophysiology: Large numbers of blood cells and plasma volume are lost. Clinical presentation: History and physical examination Signs and Symptoms: Hypotenstion, tachycardia, tachypnea, pallor, cool clammy skin. Patients with severe GI bleeding will vomit blood or excrete blood in stool. Diagnosis: CBC, blood pressure in hypotensive range or less than 100 mm Hg systolic, reticulocytosis, FOBT, CT scan, pregnancy test of childbearing age women Treatment: Establishing hemostasis, restoring blood volume, treating shock. Transfusion is currently only reliable way to restore blood volume. Endoscopy and colonoscopy should be done to investigate GI bleeding.

What is hemostasis?

A protective mechanism whereby the formation of a thrombus prevents excessive blood loss from the body.

The client is diagnosed with chronic renal insufficiency. Her vascular access site becomes incompetent, and the physician orders urokinase to dissolve the clot. What should the nurse do?

Administer the drug as ordered. Most anticoagulant, antiplatelet, and thrombolytic drugs may be used in clients with impaired renal function. For example, heparin and warfarin may be used in usual dosages, and thrombolytic agents (e.g., streptokinase, urokinase) may be used to dissolve clots in IV catheters or vascular access sites for hemodialysis. (less)

What is hyperbilirubinemia?

Accumulation of bilirubin the blood stream

The blood clotting time of the intrinsic pathway can be measured by the _________ _______ ________ ________.

Activated partial thromboplastin time (aPTT)

Specific types of antigens on the surface of RBC's are called __________.

Agglutinogens, Type A and Type B

Medications: antiplatelets

Alter formation of platelet plug (Aspirin) Indications - Reduce risk of recurrent TIAs or strokes - Reduce death or non-fatal MI - MI prophylaxis - Anti inflammatory - Analgesic Therapeutic effects → inhibits platelet aggregation by inhibit platelet synthesis of thromboxane A2 Adverse reactions - Bleeding, dizziness, weakness, GI, too much hypotension Nursing considerations - Assess history and physical exam and known allergy - Pregnancy and lactations - Know disorders, closed head injury, recent surgery - VITALS baseline of all pertinent blood values

After teaching a group of students about hemostatic agents, the instructor determines that the teaching was successful when the students identify which of the following as a systemic hemostatic agent?

Aminocaproic acid Aminocaproic acid is the only systemic hemostatic agent available. Absorbable gelatin is a topical hemostatic agent. Human fibrin sealant is a topical hemostatic agent. Recombinant thrombin is a topical hemostatic agent. (less)

A client with chronic kidney disease has been receiving epoetin alfa for the past 2 months. What would the nurse determine is an indicator that this therapy has been effective? A decrease in blood pressure An increase in white blood cells An increase in serum hematocrit A decrease in serum creatinine level

An increase in serum hematocrit Rationale:Epoetin alfa stimulates red blood cell production. Initial effects would be seen within 1 to 2 weeks, and the hematocrit reaches normal levels in 2 to 3 months. Test-Taking Strategy:Note the strategic word, effective. Focus on the subject, expected findings associated with this medication, and focus on the effects of epoetin alfa. Eliminate option 1 because as the hematocrit rises, blood pressure also may rise transiently. Eliminate option 4 because the medication does not have a direct effect on serum creatinine levels. Eliminate option 2 because epoetin alfa does not have an effect on white blood cells. An elevated white blood cell count could indicate that the client has an infection.

The nurse would monitor the platelet levels closely for a client receiving which of the following?

Anagrelide Anagrelide decreases the production of platelets in the bone marrow, necessitating close monitoring of platelet levels for thrombocytopenia.

_______ is a condition in which there is insufficient delivery of oxygen to the tissues caused by an inadequate number of mature, healthy RBC's in the blood.

Anemia

The health care provider orders thrombolytic agents when treating a client diagnosed with acute myocardial infarction. When specifically considering this client, which drug should the nurse keep readily available when blood flow is reestablished? Anticoagulants Antidysrhythmics Antihypertensives Antianginals

Antidysrhythmics When the thrombolytic agents are used in acute myocardial infarction, cardiac dysrhythmias may occur when blood flow is reestablished; antidysrhythmic drugs should be readily available.

______________ syndrome is a condition associated with the formation of multiple clots. Cause of disorder is unknown but there is a high prevalence in persons with autoimmune disease. Thrombosis can be precipitated by trauma, surgical procedures, use of drugs, or abrupt withdrawal or anticoagulant drugs.

Antiphospholipid

A 55-year-old man has been diagnosed with coronary artery disease and begun antiplatelet therapy. The man has asked the nurse why he is not taking a "blood thinner like warfarin." What is the most likely rationale for the clinician's use of an antiplatelet agent rather than an anticoagulant?

Antiplatelet agents are more effective against arterial thrombosis; anticoagulants are more effective against venous thrombosis.

_________ anemia is a life threatening disorder that can be caused by cancer, sepsis, or radiation exposure.

Aplastic

A patient's laboratory results reveal pancytopenia with normal-shaped cells and reduced reticulocyte percentage. Which of the following should be considered as possible causes? Folic-acid deficiency Thalassemia Aplastic anemia Hemolytic anemia

Aplastic anemia

A patient has been administered heparin to prevent thromboembolism development status postmyocardial infarction. The patient develops heparin-induced thrombocytopenia. Which of the following medications will be administered?

Argatroban (Acova) Heparin-induced thrombocytopenia may occur in 1-3% of those receiving heparin and is a very serious side effect of heparin. In this patient all heparin administration must be discontinued and anticoagulation managed with a direct thrombin inhibitor, such as argatroban. The patient is not administered vitamin K, calcium gluconate, or aminocaproic acid. (less)

A 61-year-old man is prescribed alteplase for an acute myocardial infarction. A priority nursing intervention will be to monitor for what during and after drug therapy? Gastrointestinal distress Liver failure A drop in blood pressure Arrhythmias

Arrhythmias Explanation: A patient who is on alteplase therapy to treat a myocardial infarction should be monitored for arrhythmias using a cardiac monitor. Reperfusion arrhythmias are common when blood flow to the heart has been reestablished. The patient's blood pressure should also be monitored but would not be the priority intervention. Gastrointestinal distress and liver failure are not associated with alteplase therapy.

A nurse is caring for a patient receiving the anticoagulant drug warfarin. What pre-administration assessments should the nurse perform before administering the drug to the patient?

Assess prothrombin time (PT) and INR. The nurse should assess the prothrombin time (PT) and INR before administering the anticoagulant drug warfarin to the patient. Observing for signs of thrombus formation, assessing for signs of bleeding, and monitoring for hypersensitivity reaction are the ongoing assessments performed in patients who are administered warfarin.

A nurse is caring for a patient receiving the anticoagulant drug warfarin. What pre-administration assessments should the nurse perform before administering the drug to the patient? Observe for signs of thrombus formation. Assess prothrombin time (PT) and INR. Assess for signs of bleeding. Monitor for hypersensitivity reaction.

Assess prothrombin time (PT) and INR. The nurse should assess the prothrombin time (PT) and INR before administering the anticoagulant drug warfarin to the patient. Observing for signs of thrombus formation, assessing for signs of bleeding, and monitoring for hypersensitivity reaction are the ongoing assessments performed in patients who are administered warfarin.

A nurse is charged with caring for a patient with disseminated intravascular coagulation. Which of the following will the nurse likely expect?

Assisting in managing clotting and bleeding developments in a severely ill patient

2. Your patient is being evaluated for a bleeding disorder. The physician orders an aPTT blood test. Which statement is TRUE about this coagulation test?* A. The aPTT is an important result used to assess the effectiveness of Warfarin. B. The aPTT assesses the intrinsic pathway of coagulation and common pathways. C. The aPTT is measured in milliseconds. D. The aPTT only assesses clotting factor VII

B. The aPTT assesses the intrinsic pathway of coagulation and common pathways.

7. Your patient, who is prescribed Warfarin for blood clots, has an INR of 1. As the nurse you know that this means?* A. The medication is therapeutic. B. The medication is not effective at preventing blood clots. C. The patient is at risk for bleeding. D. The patient is experiencing Warfarin toxicity.

B. The medication is not effective at preventing blood clots.

Anemia pernicious anemia

B12 deficiency

What is the shape of the RBC?

Biconcave disk with no nucleus

A patient is taking warfarin (Coumadin) after open heart surgery. The patient tells the home care nurse she has pain in both knees that began this week. The nurse notes bruises on both knees. Based on the effects of her medications and the report of pain, what does the nurse suspect is the cause of the pain?

Bleeding Correct Explanation: The main adverse effect of warfarin (Coumadin) is bleeding. The sudden onset of pain in the knees alerts the nurse to assess the patient for bleeding. Arthritis, torn medical meniscus, and degenerative joint disease could all be symptoms of knee pain, but the onset and combination of anticoagulant therapy is not an etiology of these types of injuries and disease. (less)

Anemia: blood loss

Blood loss Chronic - losing blood slowly over time, might not be noticeable to the patient ---- GI tract blood loss ---- Ulcers ---- Low RBCs →lower number of RBCs being broken down in spleen into iron, folic acid, vitamin B12 → new RBCs made in bone marrow now lack nutrients ---- Patients will be tired, pale, show signs of cerebral hypoxia, won't have enough tissue oxygenation Acute - hemorrhage would cause acute blood loss and symptoms, rapid decrease in intravascular volume resulting in hypoxia and hypovolemia, HR increases to compensate, trying to get blood to perfuse tissues, blood pressure decrease with low blood volume, RAAS system kicks in to try to increase blood volume and pressure, kidneys release erythropoietin to increase RBCs ----- s/s - orthostatic, tachycardia, hypotension ----- Treatment - IV fluids, blood transfusion (consider iron)

The effect on pH on Hgb's ability to bind oxygen is called the ________effect.

Bohr

A 30-year-old patient undergoes a dental procedure, which results in abnormally prolonged bleeding. The presence of which other symptoms would lead a physician to diagnose her with von Willebrand disease? Select all that apply. Dizziness, seizures, and vertigo Bruising easily Excessive menstrual blood loss Ischemia of the fingers and toes Bleeding from the nose, mouth, and gastrointestinal tract

Bruising easily Excessive menstrual blood loss Bleeding from the nose, mouth, and gastrointestinal tract

A 60-year-old patient experienced a sudden onset of chest pain and shortness of breath and was subsequently diagnosed with a pulmonary embolism in the emergency department. The patient has been started on an intravenous heparin infusion. How does this drug achieve therapeutic effect?

By inactivating clotting factors and thus stopping the coagulation cascade Heparin, along with antithrombin, rapidly promotes the inactivation of factor X, which, in turn, prevents the conversion of prothrombin to thrombin. Heparin does not achieve its therapeutic effect through the excretion or inhibition of vitamin K or by inhibiting platelet aggregation. (less)

A 60-year-old client experienced a sudden onset of chest pain and shortness of breath and was subsequently diagnosed with a pulmonary embolism in the emergency department. The client has been started on an intravenous heparin infusion. How does this drug achieve therapeutic effect? By promoting the rapid excretion of vitamin K by the gastrointestinal mucosa By inhibiting the action of vitamin K at its sites of action By inhibiting platelet aggregation on vessel walls and promoting fibrinolysis By inactivating clotting factors and thus stopping the coagulation cascade

By inactivating clotting factors and thus stopping the coagulation cascade Explanation: Heparin, along with antithrombin, rapidly promotes the inactivation of factor X, which, in turn, prevents the conversion of prothrombin to thrombin. Heparin does not achieve its therapeutic effect through the excretion or inhibition of vitamin K or by inhibiting platelet aggregation.

4. A patient is receiving continuous IV Heparin. In order for this medication to have a therapeutic effect on the patient, the aPTT should be? A. 0.5-2.5 times the normal value range B. 2-3 times the normal value range C. 1.5-2.5 times the normal value range D. 1-3.5 times the normal value range

C. 1.5-2.5 times the normal value range

9. A patient's PT result is 30 seconds. What conditions below could cause this result? Select all that apply:* A. None, this is a normal PT range. B. Vitamin K deficiency C. Liver disease D. Warfarin

C. Liver disease D. Warfarin

A nurse would anticipate the need for an increased dosage of warfarin if the patient was also receiving: Carbamazepine Clofibrate Amiodarone Danazol

Carbamazepine Carbamazepine decreases the anticoagulant effects of warfarin necessitating an increased dose of warfarin. Clofibrate, amiodarone, and danazol increase the bleeding effects of warfarin, necessitating a decreased dosage of warfarin.

Explain Hemolytic Uremic Syndrome.

Causes progressive renal failure, hemolytic anemia, and thrombocytopenia. Epidemiology: More common in children Etiology: Caused by ingestion of contaminated produce, water, or undercooked meat. Pathophysiology: Irritates the intestinal mucosa and causes a blood diarrheal illness. Clinical Presentation: Gastroenteritis, fever, blood diarrhea for 2-7 days. Causes abdominal pain, dehydration, fatigue, and very low urine output. Hypertension, edema, lethargy, and pallor. Diagnostic tests: Urinalysis, blood urea nitrogen and serum, peripheral blood smear, Bilirubin levels, study of kidney tissue. Treatment: Blood pressure control and dietary protein restriction. Antibiotics, plasma exchange.

Insufficient oxygen delivery to the tissues produces signs and symptoms related to:

Cellular hypoxia and lack of cell energy.

A patient with no known history presents with thrombocytopenia. Which is a likely next step?

Checklist of medications

What does a CBC measure?

Counts and evaluates size and shape of platelets. RBC's and WBC's, measures Hgb, Hct, and size and color of RBC's.

A client who was diagnosed with toxic shock syndrome (TSS) now exhibits petechiae, oozing from puncture sites, and coolness of the digits of the hands and feet. Clotting times determined for this client are prolonged. The nurse interprets these clinical signs as being most compatible with which condition? Heparin overdose Vitamin K deficiency Factor VIII deficiency Disseminated intravascular coagulopathy (DIC)

DIC (penache is almost always DIC) Rationale:TSS is caused by infection and often is associated with tampon use. The client's clinical signs in this question are compatible with DIC, which is a complication of TSS. The nurse assesses the client at risk and notifies the primary health care provider promptly when signs and symptoms of DIC are noted. Although signs of bleeding may be seen with each of the conditions listed in the incorrect options, the initial diagnosis of TSS makes DIC the logical correct option.

The nurse would anticipate administering fondaparinux to a client with which condition? DVT prophylaxis DVT treatment PE treatment myocardial infarction

DVT Prophylaxis Fondaparinux is indicated for DVT prophylaxis. Enoxaparin is used to treat DVT and in PE treatment as well as unstable angina/non--Q-wave myocardial infarction (MI).

Fondaparinux (Arixtra) is indicated for which of the following? (Choose one)

DVT prophylaxis Fondaparinux (Arixtra) is indicated for DVT prophylaxis.

A nurse is working with a woman suffering from fatigue. Which of the following supports the nurse's conclusion that the patient is suffering from iron-deficiency anemia? Select all that apply Decreased hemoglobin Macrocytic cells Hypochromic cells Decreased hematocrit Megaloblastic cells

Decreased hemoglobin Hypochromic cells Decreased hematocrit

Lack of bone marrow production of EBCs: Anemia of chronic disease

Diabetes, stroke, infection, inflammation, anemia is side effect - Decreased RBC survival time, blunted EPO response, impaired iron metabolism - Lack of iron makes response to EPO less than desirable - Signs and symptoms of anemia are present - Transfusions not recommended, EPO administration is preferred treatment bc higher in iron (higher than normal dose may be needed)

___________ thrombocytopenia can occur from multiple transfusions because blood stored for more than 24 hours has few or no platelets.

Dilutional

Explain Disseminated Intravascular Coagulation.

Disorder of both clot formation and bleeding episodes in critically ill patients. Epidemiology: Always secondary to another formidable disorder. Can double the risk of death in sepsis. Etiology: Most commonly observed in patients with sepsis and septic shock. Pathophysiology: Uncontrolled synthesis of thrombin, suppression of anticoagulant mechanisms, and abnormal fibrinolysis. Clinical Presentation: Bleeding may be present as petechia, purpura, oozing from the puncture sites. GI bleeding, bleeding from gums, spontaneous bruising, hemoptysis, hematuria, jaundice, cardiovascular shock. Diagnosis: Platelet counts are reduced, peripheral blood smear, fibrinolysis, D-Dimer test. Treatment: Replace clotting factors, transfusions of fresh frozen plasma, patentees, may be given Heparin to decrease blood coagulation, use of antifibinolyric agents, Protein C concentrates.

______________ is the most commonly acquired disorder of coagulation, often triggered by sepsis, occurs in 1% of all hospitalized patients.

Disseminated Intravascular Coagulation

In which disorder are fibrin clots formed rapidly, using up coagulation factors, resulting in increased risk of bleeding? Hemolytic-uremic syndrome Disseminated intravascular coagulation Immune thrombocytopenic purpura Primary thrombocytosis Thrombotic thrombocytopenic purpur

Disseminated intravascular coagulation

What is the major stimulus for eythropoiesis?

Hypoxia

The nurse is conducting staff in-service training on von Willebrand's disease. Which would the nurse include as characteristics of von Willebrand's disease? Select all that apply. Easy bruising occurs. Gum bleeding occurs. It is a hereditary bleeding disorder. Treatment and care are similar to that for hemophilia. It is characterized by extremely high creatinine levels. The disorder causes platelets to adhere to damaged endothelium.

Easy bruising occurs. Gum bleeding occurs. It is a hereditary bleeding disorder. Treatment and care are similar to that for hemophilia. The disorder causes platelets to adhere to damaged endothelium. Rationale: Von Willebrand's disease is a hereditary bleeding disorder characterized by a deficiency of or a defect in a protein termed von Willebrand factor. The disorder causes platelets to adhere to damaged endothelium. It is characterized by an increased tendency to bleed from mucous membranes. Assessment findings include epistaxis, gum bleeding, easy bruising, and excessive menstrual bleeding. Treatment and care are similar to measures implemented for hemophilia, including administration of clotting factors. An elevated creatinine level is not associated with this disorder. Test-Taking Strategy:Focus on the subject, assessment findings, and on the child's diagnosis. Recalling that this disorder is characterized by an increased tendency to bleed from mucous membranes will direct you to the correct options.

The client is prescribed warfarin. His INR is 5.2. At what level is this dose?

Elevated Warfarin dosage is regulated according to the INR (derived from the prothrombin [PT] time), for which a therapeutic value is between 2.0 to 3.0 in most conditions. A therapeutic PT value is approximately 1.5 times the control, or 18 seconds. (less)

A patient arrives at the clinic for follow-up for recently diagnosed folic-acid deficiency. Which of the following likely contributed to this diagnosis? Microcytic anemia Elevated mean corpuscular volume (MCV) Hypochromic anemia Glossitis

Elevated mean corpuscular volume (MCV) (In folic acid deficiency, the cells are enlarged. MCV indicates the volume of RBCs)

A client is admitted with thrombophlebitis and sent home on enoxaparin therapy. Which statement indicates a good understanding of why enoxaparin is being administered? Enoxaparin inhibits the formation of additional clots. Enoxaparin eliminates certain clotting factors. Enoxaparin decreases the viscosity of blood. Enoxaparin will dissolve the clots.

Enoxaparin inhibits the formation of additional clots. Low-molecular-weight heparins prevent the development of additional clots. They do not eliminate clotting factors. LMWHs do not prevent the blood from clotting. LMWHs do not dissolve the clot.

A patient is admitted with thrombophlebitis and sent home on enoxaparin (Lovenox). Which statement indicates a good understanding of why enoxaparin is being administered?

Enoxaparin inhibits the formation of additional clots. Low-molecular-weight heparins prevent the development of additional clots. They do not eliminate clotting factors. LMWHs do not prevent the blood from clotting. LMWHs do not dissolve the clot. (less)

In acute coronary syndrome (unstable angina, MI), platelet aggregation must be inhibited before medical or surgical treatment begins. Which medication exhibits antiplatelet effects during infusion that cease when the infusion is stopped?

Eptifibatide In eptifibatide infusion, antiplatelet effects occur during infusion and stop when the infusion is terminated.

Anemia: hemolysis

Erythrocyte destruction > replacement (autoimmune disorders, transfusion reactions) - Chills, jaundice, dark urine, enlarged spleen from RBC breakdown → bilirubin - rises to top of skin - Drugs that trigger hemolytic anemia - NSAIDs, penicillins, cephalosporins, levodopa, etc.

Blood components: RBCs

Erythrocytes, contain protein hemoglobin, hemoglobin, aids in oxygen transport - Erythropoiesis → synthesis of RBCs - Polycythemia → overproliferation of all blood cells and bone marrow, less common than anemia - Anemia → major pathological condition of RBCs - Normally last 120 days, removed by spleen and recycled - Immature RBCs called reticulocytes - Synthesis stimulated by a state of hypoxia which then stimulation kidneys to release EPO that stimulates RBCs to be produced - Splenomegaly occurs when a large amount of RBC breakdown is occurring the body - RBC synthesis: protein, iron, vitamin b12, folic acid

Explain Folic Acid Deficiency.

Etiology: Occurs in gluten induced intestinal disorders, intestinal resection surgery, Crohn's disease, and celiac disease. Pathophysiology: Folic Acid and DNA synthesis, Folic Acid and prevention of fetal neural tube defects, folic acid and prevention of cardiovascular disease. Clinical presentation: Thorough history and physical examination Signs and symptoms: Weakness, fatigue, difficulty concentrating, irritability, headache, palpitations, and shortness of breath. Physical examination: Pallor, tachycardia, cheilitis Diagnosis: Folic acid deficiency vs vit b deficiency, vitamin b assay and folic acid level, look at MMA level, tHCy level indicates that folic acid is not assisting in the metabolism of homocysteine Treatment: Oral replacement of folic acid, long term therapy

Which complication may be present with secondary thrombocytosis? Decreased platelet count High iron level High serum ferritin Excessive bleeding Decreased PT

Excessive bleeding

The ________ pathway is stimulated by trauma to a blood vessel occurring from an external injury such as a laceration factor.

Extrinsic

Blood coagulation results from the activation of one of two pathways: one pathway occurs in tissue, and the other occurs in the vascular system. What are the names of these pathways?

Extrinsic and intrinsic pathways The coagulation process results from the activation of the intrinsic or the extrinsic pathways. The intrinsic pathway is activated by an injury to a blood vessel; the extrinsic pathway is activated by an injury to tissue. (less)

1. A patient is ordered a PT/INR blood test. As the nurse you know that this blood test assesses?* A. Extrinsic pathway of coagulation and common pathways B. Intrinsic pathway of coagulation and common pathways C. Clotting factors XII, XI, IX, VIII D. Only clotting factor II (prothrombin)

Extrinsic pathway of coagulation and common pathways

For a male patient, age 50, recent bloodwork reveals the incidental finding of iron deficiency anemia. Which test should next be considered? Erythropoietin (EPO) assessment Computed tomography (CT) scan of abdomen Intrinsic factor assessment Fecal-occult blood test

Fecal-occult blood test

Which of the following is true regarding hemostasis? Platelet plug and clot formation are synonymous. Von Willebrand factor activates fibrin to fibrinogen. The intrinsic and extrinsic clotting pathways share no common points. Fibrinolysis is the destruction of clots. Coagulation is the formation of the platelet plug.

Fibrinolysis is the destruction of clots.

The clinic nurse instructs parents of a child with sickle cell anemia about the precipitating factors related to sickle cell crisis. Which, if identified by the parents as a precipitating factor, indicates the need for further instruction? Stress Trauma Infection Fluid overload

Fluid overload Rationale:Sickle cell crises are acute exacerbations of the disease, which vary considerably in severity and frequency; these include vaso-occlusive crisis, splenic sequestration, hyperhemolytic crisis, and aplastic crisis. Sickle cell crisis may be precipitated by infection, dehydration, hypoxia, trauma, or physical or emotional stress. The parents of a child with sickle cell disease would encourage fluid intake of 1.5 to 2 times the daily requirement to prevent dehydration. Test-Taking Strategy:Note the strategic words, need for further instruction. These words indicate a negative event query and ask you to select an option that is an incorrect statement. Recalling that fluids are a main component of treatment in sickle cell anemia to prevent crisis will direct you to the correct option. Remember that fluids are required to prevent dehydration.

Which would a nurse identify as inhibiting factor Xa? Heparin Warfarin Argatroban Fondaparinux

Fondaparinux Fondaparinux inhibits factor Xa and blocks the clotting cascade to prevent clot formation. Heparin and argatroban block the formation of thrombin from prothrombin. Warfarin decreases the production of vitamin K-depending clotting factors in the liver.

A nurse is caring for a patient receiving heparin therapy. The nurse instructs the patient and family that certain foods must be taken in limited quantity as they interfere with the effect of the therapy. Which of the following foods must be taken in limited quantity?

Foods rich in vitamin K The nurse should inform the patient to limit the intake of foods rich in vitamin K as they interfere with the heparin therapy. Foods rich in vitamin A, C, or D need not be taken in limitation, as they do not affect the treatment. (less)

Anemia - etiologies, s/s, diagnosis, treatment

General s/s of anemia - Fatigue, pallor, sob, tachycardic - Complete blood count, peripheral blood smear, bone marrow aspiration, EKG/EEG, nutritional status

Explain von WIllebrand Disease.

Genetic disorder transmitted as an autosomal trait. It is a protein that connects platelets to the endothelial lining of blood vessels and binds to factor VIII of the coagulation cascade. Epidemiology: Females more evident due to menstrual bleeding. Etiology/Pathophysiology: Genetic mutation on the short arm of chromosome 12. This disease functions to enhance platelet aggregation and platelet adhesion to endothelium and stops the degradation of factor VIII. Clinical Presentation: Easy bruising, excessive menstrual blood loss, bleeding from the nose, mouth, and GI tract. Diagnosis: Lab tests include PT, aPTT, factor VIII activity, Ristocetin cofactor can be use to estimate the activity in the blood. Treatment: Demopressin (a synthetic form of vasopressin which stimulates the endothelial cells to release vWF and plasminogen activator). Antifibrinolytic agents

Platelet adhesion results in the activation of the ______________, which binds to fibrinogen from the coagulation cascade and enhances further platelet aggregation.

Glycoprotein IIb/IIIa receptor

__________ is the percentage of blood that consists of RBC's.

Hematocrit (Hct)

A client with chronic kidney disease is receiving epoetin alfa. Which laboratory result would indicate a therapeutic effect of the medication? Hematocrit of 33% (0.33) Platelet count of 400,000 mm3 (400 × 109/L) White blood cell count of 6000 mm3 (6.0 × 109/L) Blood urea nitrogen level of 15 mg/dL (5.25 mmol/L)

Hematocrit of 33% (0.33) Rationale:Epoetin alfa is synthetic erythropoietin, which the kidneys produce to stimulate red blood cell production in the bone marrow. It is used to treat anemia associated with chronic kidney disease. The normal hematocrit level is Male: 42% to 52% (0.42 to 0.52); Female: 37% to 47% (0.37 to 0.47). Therapeutic effect is seen when the hematocrit reaches between 30% and 33% (0.30 and 0.33). The normal platelet count is 150,000 to 400,000 mm3 (150 to 400 × 109/L). The normal blood urea nitrogen level is 10 to 20 mg/dL (3.6 to 7.1 mmol/L). The normal white blood cell count is 5000 to 10,000 mm3 (5 to 10 × 109/L). Platelet production, white blood cell production, and blood urea nitrogen do not respond to erythropoietin.

The process by which all blood cells are formed in the bone marrow is called __________.

Hematopoiesis

A woman believes that she has been diagnosed with anemia. Which lab value does not support this diagnosis? Hematocrit of 38% Hemoglobin levels of 18 grams per deciliter Red blood cell total count of 2.6 million per microliter The woman's hematocrit, hemoglobin, and red blood cell count support the diagnosis of anemia

Hemoglobin levels of 18 grams per decilite

A woman believes that she has been diagnosed with anemia? Which lab value does not support this diagnosis?

Hemoglobin levels of 18 grams per deciliter

A ___________ is an inherited disorder of the structure of the Hgb molecule that can lead to destruction of the RBC.

Hemoglobinopathy

Which genetic clinical condition will likely, over the course of the client's lifetime, require the pharmaceutical introduction of clotting factors to assure the client's safety? Cystic fibrosis Hemophilia Leukemia Diabetes

Hemophilia Hemophilia is a genetic lack of clotting factors that leaves the patient vulnerable to excessive bleeding with any injury. Treatment of classic hemophilia with antihemophilic factor provides temporary replacement of clotting factors to correct or prevent bleeding episodes or to allow necessary surgery. Bone marrow disorders are disorders in which platelets are not formed in sufficient quantity to be effective. Neither diabetes nor cystic fibrosis is treated with antihemophilic agents.

What are the most commonly known inherited disorders of coagulation?

Hemophilia A, Hemophilia B, von WIllebrand disease

The physiological process that stops bleeding at the site of an injury is called ___________.

Hemostasis

_________ acs as an anticoagulant by activating antithrombin, the body's natural clot-dissolving substance. Its acts to prevent clot formation or limit the extension of clot, but it does not dissolve clots. It can be administered through an IV or subcutaneously.

Heparin

A client with a history of clot formation is scheduled for bowel resection due to colorectal cancer. What anticoagulant agent will be administered prophylactically? acetylsalicylic acid heparin warfarin streptokinase

Heparin Prophylactically, low doses of heparin are given to prevent thrombus formation in clients having major abdominal surgery. Acetylsalicylic acid is not used to prevent thrombus in clients having major abdominal surgery. Warfarin takes several days for therapeutic effects to occur; thus it is not used prophylactically to prevent thrombus in a client with abdominal surgery. Streptokinase promotes thrombolysis and is not used to prevent thrombus.

The nurse is preparing to administer protamine emergently to a client per instructions from the health care provider. The nurse concludes this is necessary due to an adverse reaction to which drug? Clopidogrel Heparin Alteplase Warfarin

Heparin An overdosage of any anticoagulant may result in uncontrolled bleeding in the client. In most cases, discontinuation of the drug is usually sufficient to correct overdosage; however, if the bleeding is severe there are antidotes. Protamine is used to treat overdose of heparin and low-molecular-weight heparins (LMWHs). Vitamin K is used to treat the overdosage of warfarin. Alteplase and clopidogrel do not require antidotes.

A client with a history of clot formation is scheduled for bowel resection due to colorectal cancer. What anticoagulant agent will be administered prophylactically? acetylsalicylic acid heparin warfarin streptokinase

Heparin Prophylactically, low doses of heparin are given to prevent thrombus formation in clients having major abdominal surgery. Acetylsalicylic acid is not used to prevent thrombus in clients having major abdominal surgery. Warfarin takes several days for therapeutic effects to occur; thus it is not used prophylactically to prevent thrombus in a client with abdominal surgery. Streptokinase promotes thrombolysis and is not used to prevent thrombus.

A female client is started on warfarin therapy. The client asks the nurse why she is no longer on heparin therapy. What differentiates heparin from warfarin? Heparin causes more serious adverse effects than warfarin. Heparin is an anticoagulant, while warfarin is a thrombolytic. Heparin is administered parenterally, while warfarin is administered orally. Heparin is used to treat thrombosis, while warfarin treats heart attacks.

Heparin is administered parenterally, while warfarin is administered orally. Heparin is administered parenterally, while warfarin is administered orally. Warfarin is administered after heparin therapy to complete treating a thrombus or embolism. Clients receiving heparin therapy will begin taking warfarin before they discontinue heparin. This overlap allows the warfarin to reach a therapeutic level before heparin is discontinued. However, both drugs are used to treat thrombosis and thromboembolic disorders and have equally serious adverse effects, such as bleeding. Anticoagulants, such as warfarin and heparin, prevent new clot formation or extension of an existing clot, while thrombolytics dissolve existing blood clots.

Ms. Adams is started on warfarin therapy. The patient asks you why she is not on heparin therapy any longer. What differentiates heparin from warfarin?

Heparin is administered parenterally, while warfarin is administered orally. Heparin is administered parenterally, while warfarin is administered orally. Warfarin is administered after heparin therapy to complete treating a thrombus or embolism. Patients receiving heparin therapy will begin taking warfarin before they discontinue heparin. This overlap allows the warfarin to reach a therapeutic level before heparin is discontinued. However, both the drugs are used to treat thrombosis and thromboembolic disorders and have equally serious adverse effects, such as bleeding. Anticoagulants, such as warfarin and heparin, prevent new clot formation or extension of an existing clot, while thrombolytics dissolve existing blood clots. (less)

Explain Thalassemia.

Heredity blood disorder Etiology: Caused by a genetic defect. In alpha thalassemia, an alpha chain is missing. In beta thalassemia, a beta chain is absent. An autosomal-recessive inheritance pattern similar to that seen in sickle cell disease. Pathophysiology: One of the major polypeptide chains of the Hgb structure is deficient, leading to reduced Hgb synthesis and decreased RBC production. Clinical Presentation: Fatigue, weakness, pallor, exercise intolerance, bones become weak, enlarged, and distorted. Spleen and liver become filled with dead RBC's. The myocardium, liver, kidney, and pancreas can be affected by iron overload. Diagnosis: CBC, Hgb electrophoresis Treatment: Splenectomy, folic acid supplementation, gene therapy, bone marrow stem cell transplants

The nurse is monitoring a client who is receiving epoetin alfa for adverse effects of the medication. Which finding indicates a side/adverse effect? Diarrhea Depression Bradycardia Hypertension

Hypertension Rationale:Epoetin alfa generally is well tolerated. The most significant adverse effect is hypertension, and its use is contraindicated in uncontrolled hypertension. Occasionally a tachycardia may occur as a side effect. This medication also may cause an improved sense of well-being. Test-Taking Strategy:Focus on the subject, side/adverse effects of epoetin alfa. Think about the action and use of this medication. Knowledge regarding the significant adverse effect associated with epoetin alfa is needed to answer this question. Remember that hypertension can occur with this medication.

Hemophilia B is caused by a lack of factor:

IX

_______________ is a disorder in which the immune system specifically targets platelets, can be triggered by a wide variety of medications and has an incidence of 66 cases per million persons per year.

Immune thrombocytopenic pupura

A patient is given tissue plasminogen activator. Which of the following will result from this treatment?

Increased firinolysis

A patient is able to produce only limited amounts of erythropoietin. What does the nurse expect to observe? Polycythemia Increased hemolysis Hyperbilirubinemia Reduced hematocrit

Increased hemolysis

Clotting disorders & risk factors (i.e. A. fib, DVT, PE)

Increased platelet number and activity ----- If too many platelets they can bleed if platelets are not doing what they are supposed to ----- Causes: Splenectomy, Myeloproliferative disorder ----- From endothelial injury and sluggish blood flow ---------- Smoking, hyperlipidemia, diabetes Increased Coagulation Activity → increase in procoagulation factor or decrease in anticoagulation ----- Stasis of blood flow ------------- Immobile, sedentary, atrial fibrillation ------------- DVT: deep vein thrombosis that may travel to lungs and may cause pulmonary embolism ----- Increase coagulation factors ---------- Estrogen: Women age 35 years and older may need to consider other alternatives to oral contraceptives ---------- Cancer: tumor cells may secrete prothrombotic factors

A 59-year-old patient is on warfarin therapy. On follow-up visits to the clinic, the nurse will assess the patient for which of the following?

Intake of vitamin K It is important to assess the patient's usual vitamin K intake because warfarin interferes with the synthesis of vitamin K-derived clotting factors. Increases in vitamin K intake will interfere with the action of warfarin if the increase in intake occurs after the warfarin dosage has been titrated. In addition, a deficiency of vitamin K can increase the risk of bleeding. The patient's sugar intake or the presence of a skin-related or breathing disorder does not affect the administration of warfarin. (less)

Medications: Anticoagulants

Interfere with clotting cascade and thrombin formation (ex. heparin) Indications - Prevention /treatment of venous thrombosis and PE - Prevention of clotting in blood samples and heparin lock sets - Heparin in AV fistulas Therapeutic effects → inhibits thrombosis and clot production by blocking prothrombin to thrombin and fibrinogen to fibrin Adverse effects → hair loss, bruising, chills, fever, osteoporosis, suppressed renal function Nursing considerations → be smart

A 59-year-old client with vascular disorders is prescribed cilostazol. The nurse instructs the client that it is for the treatment of what condition? Deep-vein thrombosis Pulmonary embolism Intermittent claudication Venous stasis

Intermittent Claudication This drug is intended for the treatment of intermittent claudication, which is an arterial disorder. Pulmonary embolism, deep-vein thrombosis, and venous stasis are all venous disorders.

A 59-year-old client with vascular disorders is prescribed cilostazol. The nurse instructs the client that it is for the treatment of what condition?

Intermittent claudication This drug is intended for the treatment of intermittent claudication, which is an arterial disorder. Pulmonary embolism, deep-vein thrombosis, and venous stasis are all venous disorders.

Individuals on anticoagulants who sustain trauma or head injury should be thoroughly assessed for signs of:

Intracranial or internal bleeding

A patient is to receive abciximab. The nurse would expect to administer the drug by which route?

Intravenous Abciximab is administered intravenously.

What is needed in the diet to synthesize Hgb in RBC's and myoglobin?

Iron

The nurse is assessing a dark-skinned client for signs of anemia. The nurse would focus the assessment on which structures? Select all that apply. Lips Tongue Earlobes Conjunctiva Mucous membranes

Lips Conjunctiva Mucous membranes Rationale, Strategy, Tip Rationale:Changes in skin color can be difficult to assess in the dark-skinned client. Color changes are most easily seen in areas of the body where the epidermis is thin and in areas where pigmentation is not influenced by exposure to sunlight. The nurse needs to assess the lips, conjunctiva, and oral mucous membranes for signs of anemia in the dark-skinned client. Signs of anemia are less easily observed in the tongue and earlobes. Test-Taking Strategy:Focus on the subject, assessment of a dark-skinned client. Recall that changes in skin color can be difficult to detect in the dark-skinned client. Recalling that areas of the body where the epidermis is thin and changes in pigmentation are not influenced by sun exposure will lead you to select lips, conjunctiva, and mucous membranes as correct answers.

Thrombopoietin is synthesized by the _________.

Liver

The nurse is assessing for the presence of pallor in a dark-skinned client suspected of having anemia. What finding would the nurse look for? A yellow tinge to the skin Bluish discoloration of the skin Loss of normal red tones in the skin An ashen-gray appearance to the skin

Loss of normal red tones in the skin Rationale:In dark-skinned clients, pallor results in the loss of normal red tones in the skin. A yellow-tinged skin could indicate jaundice. Bluish discoloration of the skin and an ashen-gray color could indicate cyanosis and circulatory compromise. Test-Taking Strategy:Options 2 and 4 can be eliminated because they are comparable or alike. Regarding the remaining options, focus on the words pallor and dark-skinned to direct you to the correct option.

A male patient is experiencing chronic renal failure and decreased synthesis of erythropoietin. Which of the following related to red blood cells may the nurse expect?

Low red blood cell number

A male patient is experiencing chronic renal failure and decreased synthesis of erythropoietin. Which of the following related to red blood cells may the nurse expect? Hematocrit of 45% Low red blood cell number Polycythemia Vitamin B12 deficiency anemia

Low red blood cell number

A male patient is experiencing chronic renal failure and decreased synthesis of erythropoietin. Which of the following related to red blood cells may the nurse expect? Hematocit of 45% Low red blood cell number Polycythemia Vitamin B12 deficiency anemia

Low red blood cell number --> erythropoietin stimulation red blood cell formation, if erythropoietic is lacking, than red blood cell # will be reduced Hematocrit - lack of an erythpoietin would cause lower-than-normal blood cell levels, thus reducing hematocrit Polycythemia - elevation in red blood cells, a lack of erythropoietin would cause reduced red blood cell numbers Vit B12 deficiency anemia

The black box warning associated with warfarin concerns its risk of causing what condition?

Major or fatal bleeding The FDA has issued a black box warning for warfarin due to its risk of causing major or fatal bleeding.

A patient with sickle cell anemia, having been hospitalized with a vaso-occlusive crisis, is being discharged. Which instructions will be helpful for the patient?

Make certain you have adequate hydration and rest, and avoid stressors when possible

A patient with sickle cell anemia, having been hospitalized with a vaso-occlusive crisis, is being discharged. Which instructions will be helpful for the patient? The number one recommendation is to increase your activity level. Do not overreact. Sickle cell anemia is not a severe disease and should not require hospitalization. Make certain you have adequate hydration and rest, and avoid stressors when possible. Use caution because your blood has limited ability to clot in a small cut may result in significant bleeding.

Make certain you have adequate hydration and rest, and avoid stressors when possible.

A patient with sickle cell anemia, having been hospitalized with vaso-occlusive crises, is being discharged. Which instructions will be helpful for the patient? The number one recommendatiion is to increase your activity level Do not overreact. Sickle cell anemia is not a severe disease and should not require hospitalization Make sure you have adequate hydration and rest, and avoid stressors when possible Use caution because your blood has limited availability

Make sure you have adequate hydration and rest, and avoid stressors when possible

What does aPTT measure?

Measures time to clot, evaluates the intrinsic pathway of coagulation cascade

The nurse is providing dietary teaching for a client with a diagnosis of chronic gastritis who is at risk for vitamin B12 deficiency. The nurse instructs the client to include which foods rich in vitamin B12 in the diet? Select all that apply. Meat Corn Liver Apples Bananas

Meat Liver Rationale: Chronic gastritis causes deterioration and atrophy of the lining of the stomach, leading to the loss of function of the parietal cells. The source of intrinsic factor is lost, which results in an inability to absorb vitamin B12, leading to development of pernicious anemia. Clients must increase their intake of vitamin B12 by increasing consumption of foods rich in this vitamin, such as meats and liver. Test-Taking Strategy: Focus on the subject, foods rich in vitamin B12. Note that apples and bananas are comparable or alike in that they are fruits. This will help you eliminate these options first. Option 2 can also be eliminated because it is a vegetable. The remaining options are the correct options.

Platelets begin as immature cells called _________ in the bone marrow.

Megakaryotes

In folic-acid deficiency, the laboratory clinician is looking for cells of which shape or color?

Megaloblastic

In folic-acid deficiency, the laboratory clinician is looking for cells of which shape or color? Normocytic Megaloblastic Microcytic Hypochromic

Megaloblastic

In folic-acid deficiency, the laboratory clinician is looking for cells of which shape or color? Normocytic Megaloblastic Microcytic Hypochromic

Megaloblastic --> folic-acid deficiency results in larger-than-normal cells, which are referred to as megaloblastic

What are the five specific signs and symptoms of Thrombotic Thrombocytopenic Purpura?

Microangiopathic hemolytic anemia, thrombocytopenic purpura, Neurological abnormalities, fever, and renal disease.

A trauma patient arrives in the emergency department with a stab wound. If a blood sample were to be looked at underneath the microscope, which type of anemia would likely be present? Megaloblastic Microcytic Sickle cell Normocytic

Normocytic (in anemia related to blood loss, cell size is not changes)

A trauma patient arrives in the emergency department with a stab would. If a blood sample were to be looked at underneath a microscope, which type of anemia would be would likely be present? Megaloblastic Microcytic Sickle cell Normocytic

Normocytic - in anemia related to blood loss, cell size is not changed

A female's blood analysis returns with the following: hematocrit at 44% and hemoglobin at 14 grams per deciliter. Which of the following should the nurse discuss with the patient? Nothing in regards to hematocrit and hemoglobin. These are normal values. The patient may want to consider iron supplementation. The patient may be experiencing excessive menstrual blood loss. The patient needs better hydration as hematocrit is too high.

Nothing in regards to hematocrit and hemoglobin. These are normal values.

A patient needs a blood transfusion. The patient has O+ blood. Which of the following blood types can the patient receive in a transfusion? Select all that apply. A+ AB+ O+ O- AB-

O+ O-

The physician discovers a clot in the client's left lower leg. The physician prescribes anticoagulant drugs to prevent formation of new clots and to achieve which other effect?

Prevent extension of clots already present Anticoagulant drugs are given to prevent formation of new clots and extension of clots already present.

Explain aplastic anemia.

Occurs when bone marrow fails, resulting in hypo cellular bone marrow and pancytopenia. Etiology: In bone marrow failure, early forms of hematopoietic cells are greatly diminished. Clinical presentation: Petechia, bleeding gums, easy bruising, nose bleeds, heavy menstrual bleeding. Diagnosis: CBC which will reveal pancytopenia. Reticulocyte, WBC, platelet count. Treatment: Blood transfusions, bone marrow transplant, prophylactic antibiotics, immunosuppressive therapy if patient older than 60

Explain Anemia of Chronic Blood loss.

Occurs when the patient endures a slow, gradual blood loss via the GI tract or excessive monthly menstrual loss. Risk Factors: Gastric ulceration, inflammatory bowel disease, colon cancer, chronic use of aspirin or NSAIDs. Pathophysiology: The RBC's are the largest iron depot in the body. When RBC's are lost, iron is lost and not recycled in the body. Slow loss of RBC's gradually depletes the body's iron stores. Clinical Presentation: Obtain list of medications, overuse of aspirin or NSAIDs, frequent use of over the counter antacids, proton pump inhibitors, or acid suppressants. History: Dark stools, past medical/surgical history, excessive menstrual bleeding, smoking, caffeine, alcohol use. Signs and symptoms: Pallor, weakness, exercise intolerance, and fatigue. Esophagitis and gastric ulceration. Diagnosis: CBC demonstrates MCHC anemia, blood tests, endoscopy, colonoscopy, FOBT, possible pregnancy test Treatment: Gastric ulcer medications or inflammatory bowel treatment, iron replacement treatment, blood transfusion when Hgb falls below 7g/dL.

A client is to receive clopidogrel. The nurse would expect to administer this agent by which route?

Oral Clopidogrel is administered orally.

Heparin

Oral/IV Anticoagulant

What values are used to monitor the effectiveness of warfarin therapy?

PT and INR Warfarin dose is regulated according to the international normalized ratio (INR), which is based on prothrombin time (PT). PT and INR are assessed daily until a stable daily dose is reached. Thereafter, PT and INR are determined every few weeks for the duration of warfarin therapy. (less)

A client is taking warfarin to prevent clot formation related to atrial fibrillation. How are the effects of the warfarin monitored? RBC aPTT PT and INR platelet count

PT and INR Explanation: The warfarin dose is regulated according to the INR. The INR is based on the prothrombin time. The red blood cell count is not indicative of warfarin dosage. The aPTT is utilized to determine heparin dose. The platelet count is required to determine warfarin dose.

A nursing student has a question about prothrombin (PT) and activated partial thromboplastin time (aPTT) values. Which of the following responses show that the student has correct understanding?

PT and aPTT, when increased, increase the risk for bleeding

You are caring for a patient who is taking Warfarin (Coumadin) to prevent venous thrombosis. You should explain that taking the drug requires daily blood samples to monitor of which of the following laboratory tests? a. Fibrinogen b. PT/INR c. Platelets d. aPTT

PT/INR PT - prothrombin time INR - international normalized ratio At the start of warfarin therapy, you should monitor PT and INR daily and adjust the dosage to maintain an INR of 2 to 3. Monitor the patient for indications of bleeding such as abdominal pain, black tarry stools, or nosebleeds. aPTT - activated partial thromboplastin time; parameter to assess every 4 to 6 hrs initially and then daily for patients receiving continuous IV heparin.

Laboratory values reveal the following: hemoglobin of 19 grams per deciliter and hematocrit of 53%. Which of the following correctly interprets these values? Select all that apply. Red blood cell (RBC) loss is increased. Hemoglobin is normal while hematocrit is less than normal. Percentage of blood volume that is plasma is reduced. Anemia has developed. Both hemoglobin and hematocrit levels are elevated.

Percentage of blood volume that is plasma is reduced. Both hemoglobin and hematocrit levels are elevated.

After reviewing the drugs that may interfere with warfarin, the students indicate that they need additional study when they identify what as requiring a dosage increase in the warfarin? Clofibrate Quinidine Phenytoin Cefoxitin

Phenytoin When combined with phenytoin, warfarin leads to a decrease in anticoagulant effect, which would necessitate an increase in the dosage of the warfarin. Clofibrate, quinidine, and cefoxitin increase the risk of bleeding with warfarin; thus, a decreased dose of warfarin would be indicated.

After reviewing the drugs that may interfere with warfarin, the students indicate that they need additional study when they identify which of the following as requiring a dosage increase in the warfarin?

Phenytoin When combined with phenytoin, warfarin leads to a decrease in anticoagulant effect, which would necessitate an increase in the dosage of the warfarin. Clofibrate, quinidine, and cefoxitin increase the risk of bleeding with warfarin; thus, a decreased dose of warfarin would be indicated. (less)

The main enzyme responsible for fibrinolysis is ____________.

Plasmin

Please place the following in order leading to red blood cell formation. Pluripotent stem cell Erythroblast Normoblast Reticulocyte Mature erythrocytes

Pluripotent stem cell Erythroblast Normoblast Reticulocyte Mature erythrocytes

An overabundance of RBC's is called ___________.

Polycythemia

A male patient is receiving heparin by continuous intravenous infusion. The nurse will instruct the patient and family members to report which of the following should it occur?

Presence of blood in urine or stools The nurse should instruct the patient and family members to report the presence of blood in urine or stools and any bleeding from the gums, nose, vagina, or wounds. The anticoagulation properties of heparin can sometimes result in abnormal bleeding. Sleepiness, drowsiness, skin rash, and dizziness are not commonly identified adverse effects of the drug. (less)

Ms. Kern is treated with heparin therapy for a pulmonary embolism. As the nurse caring for Ms. Kern, you understand that heparin acts by which mechanism?

Prevents the extension of the clot that has formed in lungs Heparin interferes with the final steps of the clotting cascade. It is used to prevent the extension of a blood clot, particularly in patients with DVT or PE. It is also used prophylactically in patients with short-term increased risk of thrombus formation, such as in the postoperative period after a total hip replacement. (less)

___________ thrombocytosis occurs in the bone marrow. Cause is unknown, but the increased number of platelets enhances the risk of clot formation.

Primary

____________ Polycythemia occurs when there is an excess of all blood cell types- RBC's, WBC's, and platelets - in the bone marrow. Rare and more often seen in older adults over the age of 60. Occurs more often in men than women.

Primary

Polycythemia - Primary

Primary aka polycythemia vera - Rare, proliferation of all cells, increased clot risk; possibly a chromosomal abnormality - Neuro symptoms cause slow blood - Epo level is low → too many RBCS - -------Treatment is periodic phlebotomy, blood thinners, bone marrow transplant, splenectomy -------- Thickens blood, flows really slow through vessels - Risk for heart attack and stroke - Hematocrit very high up to 60%

Which of the following may increase the risk of clotting? Hemophilia diagnosis Use of heparin Thrombocytopenia Von Willebrand disease Primary thrombocytosis

Primary thrombocytosis

A nurse is caring for a 64-year-old female patient who is receiving IV heparin and reports bleeding from her gums. The nurse checks the patient's laboratory test results and finds that she has a very high aPTT. The nurse anticipates that which of the following drugs may be ordered?

Protamine sulfate If a patient who receives IV heparin is found to be highly anticoagulated, protamine sulfate may be prescribed. Protamine sulfate, which is a strong base, reacts with heparin, which is a strong acid, to form a stable salt, thereby neutralizing the anticoagulant effects of heparin. Protamine sulfate does not produce the same effects for coumadin, alteplase, or ticlopidine. (less)

A client exhibits signs and symptoms of heparin overdose. The nurse would anticipate administering which of the following?

Protamine sulfate Protamine sulfate is the antidote for heparin overdose. Vitamin K is the antidote for warfarin overdose. Urokinase is a thrombolytic. Drotrecogin alfa is a C reactive protein that has anticoagulant effects. (less)

Substances needed for adequate synthesis of healthy RBC's include:

Protein, iron, vitamin B12, and folic acid.

The clotting time of the extrinsic pathway can be measured by the __________ time diagnostic test.

Prothrombin

Which of the following laboratory tests examine clotting time? Select all that apply. Parathyroid hormone Prothrombin time Aspartate aminotransferase Alanine aminotransferase Activated partial thromboplastin time

Prothrombin time Activated partial thromboplastin time

Blood components

RBC, HgB, Hct, Iron, Plasma MCV - mean corpuscular volume - size of RBC; MCH - mean corpuscular hemoglobin - mass of RBC; MCHC - mean corpuscular hemoglobin concentration - concentration of HgB (effects color)

Explain essential thrombocytosis.

Rare chronic disorder of bone marrow. It is a disorder of mggkaryote proliferation that increases the number of circulating platelets. Causes a platelet count greater than 600,000/uL. Epidemiology: More frequent in older patients, with a median age at diagnosis of 60 years. Death occurs from clotting episodes that lead to myocardial infarction or stroke. Etiology/Pathophysiology: No known etiology, excess number of platelets, some show less ability to aggregate. Clinical Presentation: Clots in small or large blood vessels , headache, paresthesias of fingers and toes, ischemia of fingers and toes, unsteadiness, vertigo, dizziness, syncope, seizures, dysarthria. GI tract, gums, urinary tract, joints, and brain are susceptible to bleeding. Diagnosis: CBC count, shows excessive number of platelets. Bone marrow biopsy shows magkaryotic hyperplasia. Blood chemistry reveals elevated uric acid levels. CT scan revels splenomegaly. Treatment: Medications, low dose aspirin.

Anemia: Thalassemia

Rare, chipmunk cheeks

The nurse is providing instructions to the parent of a child with iron-deficiency anemia about the administration of a liquid oral iron supplement. Which statement, if made by the parent, indicates an understanding of the administration of this medication? "I would give the iron with food." "I can mix the iron with cereal to give it." "I would add the iron to the formula in the baby's bottle." "I need to use a medicine dropper and place the iron near the back of the throat."

Rationale:An oral iron supplement needs to be administered through a straw or a medicine dropper placed at the back of the mouth because it will stain the teeth. The parents need to be instructed to brush or wipe the teeth after administration. Iron is administered between meals because absorption is decreased if there is food in the stomach. Iron requires an acidic environment to facilitate its absorption in the duodenum. Test-Taking Strategy:Focus on the subject, oral iron supplements. Eliminate options 2 and 3 first because they are comparable or alike and because medication would not be added to food or formula. Also, note the word liquid in the question. This would assist in recalling that liquid iron stains teeth.

The nurse is caring for a client who was just admitted to the hospital for the treatment of iron overload. The nurse anticipates that the primary health care provider will prescribe which medication to treat the iron overload? Terbinafine Granisetron Ketoconazole Deferoxamine

Rationale:Deferoxamine is a medication used to treat iron overload. Granisetron is an antiemetic. Ketoconazole and terbinafine are antifungal medications. Test-Taking Strategy:Focus on the subject, iron overload. Recall knowledge of the classifications of the medications identified in the options. Eliminate options 1 and 3 first because they are comparable or alike and are antifungal medications. Regarding the remaining options, you must know that granisetron is an antiemetic and that deferoxamine is the medication used to treat iron overload.

Epoetin alfa by the subcutaneous route is prescribed for a client. What is the most appropriate nursing action? Shake the vial before use. Freeze the medication before use. Refrigerate the medication until used. Obtain syringes with 1½-inch (3.8 cm) needles for administration.

Rationale:Epoetin alfa would be refrigerated at all times. The bottle would not be shaken, and the medication would not be frozen because this will affect the chemical composition. Syringes with a ⅚-inch (1.5-cm) needle are used for subcutaneous injection. A 1½-inch (3.8-cm) needle may be used for intramuscular injection. Test-Taking Strategy:Note the strategic words, most appropriate. Focus on the subject, procedures related to the administration of epoetin alfa. Noting that the medication is prescribed by the subcutaneous route will assist in eliminating option 4. Note that the correct option and option 2 identify opposite actions. This would provide you with the clue that one of these options may be the correct one.

The nurse is assessing a dark-skinned client for the presence of petechiae. Which body area is the best for the nurse to check in this client? Sclera Oral mucosa Soles of the feet Palms of the hands

Rationale:In a dark-skinned client, petechiae are best observed in the conjunctivae and oral mucosa and in areas of lighter melanization such as the abdomen and buttocks. Jaundice would best be noted in the sclera of the eye. Cyanosis is best noted on the palms of the hands and soles of the feet. Test-Taking Strategy:Note the strategic word, best. Eliminate options 3 and 4 first because they are comparable or alike and are similar body areas. Regarding the remaining options, focusing on the word petechiae will direct you to option 2.

What is the hemoglobin from erythrocytes?

Red blood cells break down to heme and globin -> Heme breaks down into iron and porphyrin -> Biliverdin -> bilirubin -> jaundice

Laboratory studies are performed for a child suspected to have iron-deficiency anemia. The nurse reviews the laboratory results, knowing that which result indicates this type of anemia? Elevated hemoglobin level Decreased reticulocyte count Elevated red blood cell count Red blood cells that are microcytic and hypochromic

Red blood cells that are microcytic and hypochromic Rationale: In iron-deficiency anemia, iron stores are depleted, resulting in a decreased supply of iron for the manufacture of hemoglobin in red blood cells. The results of a complete blood cell count in children with iron-deficiency anemia show decreased hemoglobin levels and microcytic and hypochromic red blood cells. The red blood cell count is decreased. The reticulocyte count is usually normal or slightly elevated. Test-Taking Strategy:Focus on the subject, laboratory findings. Eliminate options 1 and 3 first, knowing that the hemoglobin and red blood cell counts would be decreased. From the remaining options, select the correct option over option 2 because of the relationship between anemia and red blood cells. Rationale

What does Fibrinogen measure?

Reflects clotting ability and activity

What do fibrin degradation products measure?

Reflects clotting activity.

The nurse is reviewing a pediatrician's prescriptions for a child with sickle cell anemia who was admitted to the hospital for the treatment of vaso-occlusive crisis. Which prescriptions documented in the child's record would the nurse question? Select all that apply. Restrict fluid intake. Position for comfort. Avoid strain on painful joints. Apply nasal oxygen at 2 L/minute. Provide a high-calorie, high-protein diet. Give meperidine, 25 mg intravenously, every 4 hours for pain.

Restrict fluid intake Give meperidine, 25 mg intravenously, every 4 hours for pain. Rationale:Sickle cell anemia is one of a group of diseases termed hemoglobinopathies, in which hemoglobin A is partly or completely replaced by abnormal sickle hemoglobin S. It is caused by the inheritance of a gene for a structurally abnormal portion of the hemoglobin chain. Hemoglobin S is sensitive to changes in the oxygen content of the red blood cell; insufficient oxygen causes the cells to assume a sickle shape, and the cells become rigid and clumped together, obstructing capillary blood flow. Oral and intravenous fluids are an important part of treatment. Meperidine is not recommended for a child with sickle cell disease because of the risk for normeperidine-induced seizures. Normeperidine, a metabolite of meperidine, is a central nervous system stimulant that produces anxiety, tremors, myoclonus, and generalized seizures when it accumulates with repetitive dosing. The nurse would question the prescription for restricted fluids and meperidine for pain control. Positioning for comfort, avoiding strain on painful joints, oxygen, and a high-calorie and high-protein diet are also important parts of the treatment plan. Test-Taking Strategy:Focus on the subject, identifying the prescriptions that need to be questioned and on the pathophysiology that occurs in sickle cell disease. Recalling that fluids are an important component of the treatment plan will assist in identifying that a fluid restriction prescription would need to be questioned. Also, recalling the effects of meperidine will assist in identifying that this prescription needs to be questioned.

The markedly higher percentage of reticulocytes in the bloodstream is called _________.

Reticulocytosis

The nurse is performing an assessment on a client with a diagnosis of anemia that developed as a result of blood loss after a traumatic injury. The nurse would expect to find which sign or symptom in the client as a result of the anemia? Your Answers: Bradycardia Muscle cramps Increased respiratory rate Shortness of breath with activity

SOB Rationale:The client with anemia is likely to experience shortness of breath and complain of fatigue because of the decreased ability of the blood to carry oxygen to the tissues to meet metabolic demands. The client is likely to have tachycardia, not bradycardia, as a result of efforts by the body to compensate for the effects of anemia. Muscle cramps are an unrelated finding. Increased respiratory rate is not an associated finding. Test-Taking Strategy:Focus on the subject, assessment findings in anemia. Recalling that anemia causes a reduction in oxygen-carrying capacity will assist in directing you to the correct option.

In ___________ hemostasis, coagulation factors undergo a process known as the coagulation cascade, which leads to fibrin formation, and eventually a finished fibrin clot or thrombus.

Secondary

___________ Polycythemia, also called erythrocytosis, is caused by prolonged hypoxia as a compensatory effort by the body to improve oxygen delivery.

Secondary

A patient's mother worries about a vaso-occlusive crisis for her son, who desires to play a team sport. Which of the following conditions does the son likely have?

Sickle cell anemia

Which of the following is correct information to share with a parent whose child has been diagnosed with sickle cell anemia? Select all that apply. Sickle cell anemia is a genetic disorder. In certain conditions, your child's red blood cells, which carry oxygen, will undergo a shape change. Sickle cell anemia is not a severe disease, and very little management will be needed for this condition. We cannot be certain your child has sickle cell anemia, as there are no diagnostic tests for this disease. Your child will require a bone marrow transplant.

Sickle cell anemia is a genetic disorder. In certain conditions, your child's red blood cells, which carry oxygen, will undergo a shape change.

A young male patient is admitted in severe pain suffering from a vaso-occlusive crisis. Which condition likely underlies this development? Iron-deficiency anemia Sickle-cell anemia Aplastic anemia Pernicious anemia

Sickle-cell anemia

A patient is to receive desirudin to prevent deep vein thrombosis with an elective hip surgery. The nurse would expect this drug to be administered by which route?

Subcutaneous Desirudin is administered subcutaneously 5 to 15 minutes before the surgery and continuing for 9 to 12 days after the surgery.

Enoxaparin (Lovenox) should be administered via which route? (Choose one)

Subcutaneous injection Enoxaparin (Lovenox) is administered via subcutaneous injection.

Decreased platelet production can result from:

Suppression of bone marrow

A nurse is to speak to a patient about his recent vitamin-B12 deficiency. Which recommendations would benefit the patient? Select all that apply. Eat a vegetarian diet. Restrict folic-acid intake. Take a supplement of vitamin B12. Food choices should include meat and eggs. Get plenty of sunshine as ultraviolet (UV) light activates vitamin B12.

Take a supplement of vitamin B12. Food choices should include meat and eggs.

Ms. Main is sent home with subcutaneous heparin after a total hip replacement. What symptom would indicate a serious drug reaction?

Tarry stools Tarry stools would be an indication of gastrointestinal bleeds. The most common adverse effect of heparin is bleeding.

A patient has hemophilia. Which shows a correct understanding of this disease? Thalassemia is the result of folic acid deficiency. Thalassemia is also known as sickle cell anemia. shape. Thalassemia is a result of a genetic mutation. Thalassemia is commonly outgrown by adulthood.

Thalassemia is a result of a genetic mutation. (hemophilia can be an inherited disorder or arise from spontaneous mutation

A patient has thalassemia. What shows a correct understanding of this disease? Thalaseemia is the result of a folic acid deficiency Thalassemia is also known as sickle cell disease Thalessemia is the result of a genetic mutation Thalassemia is commonly outgrown by adulthood

Thalessemia is the result of a genetic mutation

The client is diagnosed with hepatitis A, diabetes type 1, and portal hypertension. The client develops a DVT, and the health care provider prescribes warfarin. The nurse is concerned for what reason? The client is more likely to experience bleeding. The client is less likely to achieve a therapeutic dose. The client is at risk for further liver impairment. The client is at risk for hyperglycemic episodes.

The client is more likely to experience bleeding. Warfarin is more likely to cause bleeding in clients with liver disease because of decreased synthesis of vitamin K. In addition, warfarin is eliminated only by hepatic metabolism and may accumulate with liver impairment.

A female patient with chronic obstructive pulmonary disease (COPD) presents with a hematocrit value of 48%. What is the likely explanation?

The decrease oxygenation of the blood associated with COPD has increased erythropoietin release.

A female patient with chronic obstructive pulmonary disease (COPD) presents with a hematocrit value of 48%. What is the likely explanation? The patient has primary polycythemia. The patient has aplastic anemia. The decrease oxygenation of the blood associated with COPD has increased erythropoietin release. The COPD has caused dehydration, increasing hematocrit values.

The decrease oxygenation of the blood associated with COPD has increased erythropoietin release.

A female patient with chronic obstructive pulmonary disease presents with a hematocrit value of 48%. What is the likely explanation? The patient has primary polycythemia The patient has aplastic anemia The decreased oxygenation of the blood associated with COPS has increased erythropoietin release The COPD has caused dehydration, increasing hematocrit values

The decreased oxygenation of the blood associated with COPS has increased erythropoietin release --> increased erythropoietin will increase RBC levels The patient has primary polycythemia --> primary polycythemia is very rare. This would be an unlikely cause The patient has aplastic anemia --> increased erythropoietin will increase RBC levels The COPD has caused dehydration, increasing hematocrit values

An international normal ratio (INR) value of 2.5 is reported. Which of the following is the correct interpretation?

The patient has a prolonged prothrombin time (PT).

A patient has a platelet count of 15,000 cells per microliter. Which of the following is likely?

The patient is at increased risk for bruising and petechiae

A nurse notes on the laboratory values a significant elevation in the percentage of reticulocytes. Which of the following may the nurse consider? The patient is at an increased risk for infection. The patient is synthesizing new red blood cells (RBCs) at a high level. The patient's bone marrow has been destroyed. The patient has no erythropoietin.

The patient is synthesizing new red blood cells (RBCs) at a high level.

Which of the following would be considered a topical hemostatic agent?

Thrombin Thrombin is a topical hemostatic agent. Protamine sulfate is the antidote for heparin. Pentoxifylline is a hemorrheologic agent (one that can induce hemorrhage). Urokinase is a thrombolytic agent. (less)

What is erythropoiesis?

The specific series of steps in the bone marrow that leads to the synthesis of mature RBC's.

What would be considered a topical hemostatic agent? Thrombin Protamine sulfate Pentoxifylline Urokinase

Thrombin Thrombin is a topical hemostatic agent. Protamine sulfate is the antidote for heparin. Pentoxifylline is a hemorrheologic agent (one that can induce hemorrhage). Urokinase is a thrombolytic agent.

Which would the nurse identify as the end of the intrinsic pathway? Thrombin formation Vasoconstriction Platelet aggregation Release of factor XI

Thrombin formation Thrombin formation occurs at the end of the intrinsic pathway. The first reaction to a blood vessel injury is local vasoconstriction. In addition, injury then exposes blood to the collagen and other substances under the endothelial lining of the vessel causing platelet aggregation. Release of factor XI occurs in response to activation of the Hageman factor.

____________ can result from decreased platelet synthesis in the bone marrow, increased sequestering of platelets in the spleen, or decreased platelet life span.

Thrombocytopenia

_________________ is caused by a deficiency of a metalloprotease enzyme that acts on von WIllebrand factor.

Thrombotic Thrombocytopenic Purpura

What does INR measure?

Time for patient's blood to clot compared to normal time for blood to clot.

_________ _______ ________ enzymatically changes plasminogen into plasmin. Plasmin then breaks down the clot. After the clot is dissolved, blood flow can be reestablished.

Tissue Plasminogen Activator

A client is diagnosed with iron-deficiency anemia, and ferrous sulfate is prescribed. The nurse would tell the client that it would be best to take the medication with which food? Milk Boiled egg Tomato juice Pineapple juice

Tomato juice Rationale:Ferrous sulfate is an iron preparation, and the client is instructed to take the medication with orange juice or another vitamin C-containing product or a product high in ascorbic acid to increase the absorption of the iron. Among the options presented, tomato juice is highest in vitamin C and ascorbic acid. Milk and eggs inhibit absorption of iron. Test-Taking Strategy:Note the strategic word, best. Focus on the subject, ferrous sulfate. Recalling that vitamin C and ascorbic acid increase the absorption of iron will assist in eliminating options 1 and 2.

In iron deficiency anemia, which serum factor may be high?

Total iron binding capacity

In iron deficiency anemia, which serum factor may be high? Serum iron Hematocrit Hemoglobin Total iron binding capacity

Total iron binding capacity

In iron deficiency anemia, which serum factor may be high? Serum iron Hematocrit Hemoglobin Total iron binding capacity

Total iron binding capacity --> increases when iron levels are low, indicating there is increased space available for iron to bind

The nurse is reviewing the prescriptions for a client admitted to the hospital with a diagnosis of idiopathic autoimmune hemolytic anemia. The nurse prepares the client for treatment of this disorder, understanding that which may be recommended? Select all that apply. Transfusions Splenectomy Radiation therapy Corticosteroid medication Immunosuppressive agents

Transfusions Splenectomy Corticosteroid medication Immunosuppressive agents Rationale:Idiopathic autoimmune hemolytic anemia is a decrease in the number of red blood cells due to increased destruction by the body's defense (immune) system. It is an acquired disease that occurs when antibodies form against a person's own red blood cells. In the idiopathic form of this disease, the cause is unknown. Idiopathic autoimmune hemolytic anemia is treated with corticosteroids. Other treatments that may be prescribed as necessary include transfusions, splenectomy, and, occasionally, immunosuppressive medications. Radiation therapy is not used to treat this disorder. Test-Taking Strategy:Focus on the subject, treatment for idiopathic autoimmune hemolytic anemia. Note the words may be recommended in the question. Recalling that this disorder involves the immune system will assist you in eliminating option 3.

Heparin is the anticoagulant of choice during lactation.

True Although some adverse fetal affects have been reported with its use during pregnancy, heparin does not enter breast milk, and so it is the anticoagulant of choice if one is needed during lactation.

A patient is prescribed eptifibatide (Integrilin), which inhibits platelet aggregation by preventing activation of GP IIb/IIIa receptors on the platelet surface and the subsequent binding of fibrinogen and von Willebrand factor to platelets. Which of the following syndromes are treated with eptifibatide?

Unstable angina Eptifibatide (Integrilin) inhibits platelet aggregation by preventing activation of GP IIb/IIIa receptors on the platelet surface and the subsequent binding of fibrinogen and von Willebrand factor to platelets. Eptifibatide is used for acute coronary syndromes, including unstable angina, myocardial infarction, and non-Q wave MI. Blocked carotid arteries, intermittent claudication, and hypertension are not treated with eptifibatide. (less)

What is the most common cause of impaired platelet function?

Use of aspirin and other NSAIDs

Hemophilia A is caused by a deficiency of factor:

VIII

When describing the clotting process, which step would the nurse identify as the first reaction to occur with injury to a blood vessel? Platelet aggregation Vasoconstriction Release of factor XI Thrombin formation

Vasoconstriction The first reaction to a blood vessel injury is local vasoconstriction. In addition, injury then exposes blood to the collagen and other substances under the endothelial lining of the vessel, causing platelet aggregation. Release of factor XI occurs in response to activation of the Hageman factor. Thrombin formation occurs at the end of the intrinsic pathway.

You are caring for a 73-year-old client receiving warfarin. When you perform your initial shift assessment, you observe blood in the client's urinary drainage bag. After reporting your observation to the physician, which of the following substances will you likely administer?

Vitamin K Genitourinary bleeding is an adverse effect of warfarin. Vitamin K, a hemostatic agent that controls bleeding caused by warfarin overdose, will likely be administered to this patient. (less)

A patient who has been treated with warfarin (Coumadin) after open heart surgery is involved in a car accident and has a laceration on her head. Which medication will be administered to assist in the development of clotting factors?

Vitamin K Vitamin K is the antidote for warfarin overdosage. In this case, the patient may be at the therapeutic level to control thrombus formation, but, due to the injury, it is important to control bleeding. Vitamin E is not used as an antidote for warfarin overdosage. Protamine sulfate is used as an antidote to heparin or low-molecular-weight heparin. Acetylsalicylic acid (Aspirin) is used to decrease coagulation as a preventive measure for myocardial infarction. (less)

A client who is receiving warfarin has blood in his urinary drainage bag. What medication will likely be prescribed by the health care provider? aminocaproic acid platelets protamine sulfate vitamin K

Vitamin K is the antidote for warfarin overdose. Aminocaproic acid is used to control excessive bleeding from systemic hyperfibrinolysis. Platelets are a blood product, not a medication. Protamine sulfate is the antidote for heparin therapy.

A patient is place on warfarin, which he refers to as a "blood thinner, which makes my blood more watery." Which of the following responses by the nurse is appropriate?

Warfarin does not dilute the blood but rather decreases the risk for clot formation.

porphyrin is converted to _____, which is then converted to______

biliverdin Bilirubin

A client, taking warfarin after open heart surgery, tells the home care nurse she has pain in both knees that began this week. The nurse notes bruises on both knees. Based on the effects of her medications and the report of pain, what should the nurse suspect is the cause of the pain? joint thrombosis torn medial meniscus degenerative joint disease caused by her medication bleeding

bleeding The main adverse effect of warfarin is bleeding. The sudden onset of pain in the knees alerts the nurse to assess the client for bleeding. Arthritis, torn medical meniscus, and degenerative joint disease could all be symptoms of knee pain, but the onset and combination of anticoagulant therapy is not an etiology of these types of injuries and disease.

Antibodies against platelets

bleeding disorder

Immune Thrombocytopenic Purpura (ITP)

bleeding disorder

Von Willebrand disease

bleeding disorder

Increased activation of the coagulation systemic is caused by stasis of:

blood flow, increase in procoagulation factors, or decrease in anticoagulation factors

A client who is recovering in hospital from a bilateral mastectomy has developed minor bleeding at one of the incision sites. During the process of clot formation, plasminogen will become part of a clot by which means? by binding with fibrin by binding with platelets by activating plasmin by activating factor VII

by binding with fibrin When a blood clot is being formed, plasminogen, an inactive protein present in many body tissues and fluids, is bound to fibrin and becomes a component of the clot. Plasminogen does not bind to platelets, activate plasmin, or active factor VII.

Hemolytic-uremic syndrome __________________. Select all that apply. usually results from a viral infection. causes hemolysis. causes thrombocytosis. is associated with hemophilia. can develop from exposure to Shiga-toxin.

causes hemolysis. can develop from exposure to Shiga-toxin.

Primary and secondary thrombocytosis

clotting disorder

Smoking and blood flow stasis

clotting disorder

Treatments include aspirin and thrombolytic agents

clotting disorder

Treatments include heparin and warfarin

clotting disorder

You are caring for a 76-year-old client who is receiving IV heparin 5000 units q4h. At the initiation of therapy, the patient's control activated thromboplastin time (aPTT) was found to be 35 seconds. One hour prior to the next scheduled dose, aPTT is determined to be 92 seconds. Based on this result, you will

hold the dose and contact the physician. During heparin therapy, aPTT values are typically maintained between 1.5 to 2.5 times the control value. Since an aPTT value of 92 seconds is greater than 2.5 times this patient's control value, the anticoagulation effect is too great, and the physician should be notified. (less)

Please place in the proper order the processes leading to the development of jaundice. Increased porphyrin levels Increased biliverdin levels increase in RBC break down accumulation of bilirubin in skin increased bilirubin levels

increase in RBC break down Increased porphyrin levels Increased biliverdin levels increased bilirubin levels accumulation of bilirubin in skin

A sickle cell anemia crisis leads to hemolysis of red blood cells (RBCs). Which of the following may the nurse expect to see? Decreased serum bilirubin Increased reticulate percentage Increased hematocrit Decreased likelihood of jaundice

increased reticulate percentage (Immature RBCs increase to replace lost cells)

In defective coagulation, prolonged bleeding is provoked by:

injury or trauma

Mr. Bradford is a 66-year-old smoker with a diagnosis of peripheral vascular disease who experiences lower leg pain when walking. In the last several months, his leg pain has progressed to the point where he now needs to take a rest when walking from the elevator to his apartment. Mr. Bradford has now sought care from his primary care provider, who would likely be justified in prescribing

pentoxifylline (Trental). The hemorheologic drugs, such as pentoxifylline is used to manage the symptoms of intermittent claudication from peripheral vascular disease. Using this drug improves the patient's ability to walk for longer distances without pain. Warfarin, alteplase, and ASA are not used to decrease blood viscosity and resolve the impaired perfusion associated with peripheral vascular disease. (less)

The nurse is preparing home care instructions for the parents of a 10-year-old child with hemophilia. Which sport activity would the nurse suggest for this child? Soccer Basketball Swimming Field hockey

swimming

An older adult client has been admitted to the emergency department with severe chest pain. Onset of symptoms is within the last 60 minutes. What medication would the nurse expect the health care provider to prescribe for this acute disorder? thrombolytic drugs anticoagulant drugs antiplatelet drugs direct thrombin inhibitor drugs

thrombolytic drugs The scenario suggests that the client's chest pain may be a result of a thromboembolism. The main use of thrombolytic agents is for management of acute, severe thromboembolic disease, such as myocardial infarction or pulmonary embolism since they dissolve blood clots. None of the other options can dissolve an existing blood clot.

The extrinsic pathway is also known as the:

tissue factor pathway

Which of the following statements are true in regards to anticoagulants? Select all that apply:

• Anticoagulants prevent formation of a thrombus. • Anticoagulants prevent extension of a thrombus. Explanation: Anticoagulants can prevent the formation and extension of a thrombus, but have no direct effect on an existing thrombus or reverse any of the damage from that thrombus. Although clients often refer to anticoagulants as blood thinners, they do not actually thin the blood. (less)

The client is prescribed a thrombolytic agent. The nurse understands that the purpose of this order may be to achieve which effects? (Select all that apply.)

• Dissolve thrombi • Limit tissue damage Explanation: Thrombolytic agents are used to dissolve thrombi and limit tissue damage in selected thromboembolic d

Which of the following are situations in which the nurse would hold the drug and notify the physician? Select all that apply:

• PT exceeds 1.5 times the control value • There is evidence of bleeding • INR is greater than 3.0 Explanation: The nurse should withhold the drug and contact the physician if any of the following occur: the PT exceeds 1.5 times the control value, there is evidence of bleeding, or the INR is greater than 3.0.

CH 14 Chapter Summary

• Platelet formation is stimulated by the hormone thrombopoietin, which is synthesized by the liver. • The normal platelet count is 150,000 to 450,000 cells/mcL, and a platelets life span is 7 to 10 days. • Almost one-third of all platelets reside in the spleen; when enlarged and hyperactive, the spleen can sequester up to 80% of platelets. • Numerous GP receptors (GPIIb/IIIa) on the surface of a platelet assist in clot formation. Some antiplatelet agents work as GPIIb/IIIa receptor blockers to diminish platelet aggregation. • The activated platelet surface provides the major physiological site for coagulation factor activation. • Coagulation factors are produced in the liver and require vitamin K. • Clots formed because of stasis of blood or endothelial injury are created via the intrinsic pathway. • Clots formed because of external trauma to a blood vessel are created via the extrinsic pathway. • Acquired deficiencies of coagulation are the most frequently encountered disorders in the clinical area. Causes include liver disease, vitamin K deficiency, and DIC. • The most common hereditary bleeding disorder is hemophilia. In these disorders, blood coagulation is hindered. Bleeding can be occult, as in gastrointestinal blood loss, or apparent, as in traumatic hemorrhage. • Many different drugs can cause thrombocytopenia. • In platelet dysfunction or deficiency, lesions such as petechiae may be seen on physical examination. Spontaneous bleeding can occur as bruises, nosebleeds, bleeding from the gums, or vaginal bleeding. • Hypercoagulability of the blood causes susceptibility to clotting. Clots can occlude blood vessels and cause tissue ischemia, infarction, or gangrene. Stroke, myocardial infarction, peripheral arterial disease, DVT, and pulmonary embolism are all caused by blood clots. • The INR is a standard measurement of clotting. If a blood sample has an INR of 1, this indicates normal clotting. The greater the INR, the longer it takes for the blood to clot. • For adequate anticoagulation, an INR should be maintained between 2 and 3. • The greater the patient's PT or aPTT compared with normal values, the longer it takes for the patient's blood to clot, which can increase risk for bleeding. • Aspirin is commonly used to decrease platelet aggregation. • Protamine sulfate is a heparin antagonist. • Vitamin K antagonizes warfarin. • Patients on warfarin should avoid eating excessive amounts of green leafy vegetables. • DIC, where the coagulation system dysfunctions and causes both clotting and bleeding, can occur in critically ill patients, particularly those who have sepsis.

Antiplatelet agents decrease the formation of the platelet plug by decreasing the responsiveness of the platelets to stimuli that would cause them to stick and aggregate on a vessel wall. The nurse knows that which drug is an antiplatelet agent? Select all that apply.

• ReoPro • Ticlopidine Explanation: Antiplatelet agents available for use include: abciximab (ReoPro), anagrelide (Agrylin), aspirin, cilostazol (Pletal), clopidogrel (Plavix), dipyridamole (Persantine), eptifibatide (Integrilin), sulfinpyrazone (Anturane), ticlopidine, and tirofiban (Aggrastat). Iprivask and Arixtra are anticoagulants, and Activase is a thrombolytic agent. (less)

After teaching a group of students about anticoagulants, the instructor determines that the teaching was successful when the students identify which of the following agents as interacting with warfarin that increases the risk of bleeding? Select all that apply.

• Salicylates • Ranitidine • Metronidazole • Cefoxitin Salicylates, ranitidine, metronidazole, and cefoxitin interact with warfarin resulting in the increased risk of bleeding. Barbiturates and carbamazepine interact with warfarin resulting in a decreased anticoagulant effect.


Ensembles d'études connexes

Prep U Chapter 34: Assessment and Management of Patients with Inflammatory Rheumatic Disorders

View Set

Flashcard for Chapter 6 Terms and Definitions

View Set

3/4 adrenal gland hormones and regulations

View Set

Chapter 3: Interdependence and the Gains from Trade

View Set